SlideShare a Scribd company logo
1 of 101
Pediatric Questions
AAOS 2022
Figures 1 and 2 are the standing radiographs of a healthy and active
13-year-old female who is being evaluated for an asymmetric
appearance of her back. She has no back pain or neurological
symptoms. Her first menstrual cycle was 3 months ago. Physical
examination shows appropriate motor and sensory strength and no
neurological deficits. Based on the patient's age, skeletal maturity,
and curve magnitude, what is the most appropriate treatment?
A. Physical therapy to work on core
strengthening and postural
exercises
B. Customized rigid
thoracolumbosacral orthosis
(TLSO)
C. Posterior spinal fusion surgery
D. Growing rod instrumentation
Figure 1 is the standing leg length radiograph of a 4-year-old female
who is being evaluated for tibial bowing that has worsened over the
past year. Physical examination shows significant genu varum, short
stature, and internal tibial torsion. What is the best next step in
management?
A. Observation with repeat radiographs in 6
months
B. Blood work and referral to pediatric
endocrinology
C. Bilateral lateral proximal tibia
hemiepiphysiodesis
D. Bilateral proximal tibia osteotomy
Figure 1 is the anteroposterior radiograph of the pelvis of a 12-year-
old male who weighs 98 kg and has had right knee and thigh pain
for the past 4 weeks. He has no history of recent trauma and has
been able to ambulate independently with a limp. What is the most
common physical examination finding consistent with this
pathology?
A. Positive Galeazzi sign
indicating a leg length
discrepancy
B. Foot progression angle that
is rotated internally with
gait
C. Increased external rotation
when flexing the hip to 90°
D. Painful crepitus with hip
range of motion
A 3-week-old infant is being evaluated for the bilateral foot
abnormalities shown in Figure 1. A routine course of weekly
Ponseti casting is initiated. Following 4 to 6 weeks of serial casting,
what is the most common next step in treatment?
A. Percutaneous or
open Achilles
tenotomy
B. Application of full-
time bilateral foot
abduction
orthoses
C. Open posterior
tibial tendon
transfer
D. Split anterior tibial
tendon transfer
Figures 1 and 2 are the radiographs of an 11-year-old girl who is
having right elbow pain after “trying to beat up a snowman.” She
cannot extend her elbow, has point tenderness to palpation over
the proximal ulna. Her underlying condition is associated with a
mutation in which gene?
A. Fibroblast growth
factor receptor 3
(FGFR3)
B. Diastrophic
dysplasia sulfate
transporter (DTDST)
C. COL1A1, COL1A2
D. COL2A1
Figure 1 is a full standing leg length radiograph of a 15-year-old
competitive ski racer who has a small imbalance in his pelvis. He does not
have pain or discomfort. Medical history includes a left femur fracture at 2
years of age. Physical examination shows a positive Galeazzi sign with the
left femur longer than the right. There is a 1.5-cm leg length discrepancy
with the left femur being longer than the right femur. What is the most
appropriate treatment?
A. Immediate distal femoral and proximal tibial
epiphysiodesis of the left leg
B. Use of an accommodative shoe lift
C. Right femur lengthening with external fixation or an
internal magnetic rod
D. Left femur shortening osteotomy
A 14-year-old female cross country athlete has new onset of left
ankle pain and swelling. She has no history of recent trauma. The
anteroposterior radiograph of the ankle (Figure 1) shows a small
lesion in the superior medial dome of the talus, and the coronal
ankle MRI scan (Figure 2) shows greater detail of the lesion. What is
the best next step in treatment?
A. Custom orthotic and activity modification
B. Non-weight bearing in a cast or boot
followed by physical therapy
C. Open curettage and osteochondral graft
implantation
D. Ankle arthroscopy with either retrograde
or antegrade drilling of the lesion
An olecranon fracture in a child is most likely the first presenting sign
of which of the following diseases?
A. Ehlers-Danlos
B. Osteogenesis imperfecta
C. Multiple epiphyseal dysplasia
D. Osteopetrosis
An 8-year-old male sustained the fracture shown in the
radiograph in Figure 1. What long-term problem is this
child most likely to experience?
A.Genu varum
B. Genu recurvatum
C. Procurvatum
D.Genu valgum
E.
A 2-week-old infant is being evaluated for the foot deformity
shown in Figures 1 and 2. Which of the following is the most
appropriate course of action?
A. Perform serial casting
B. Obtain a radiograph of
the leg and schedule
follow-up
C. Reassure the parents that
there is no need for
further follow-up
D. Prescribe physical
therapy for stretching
exercises and taping
In addition to shoulder dystocia and mechanically
assisted delivery, which of the following is another
common risk factor for brachial plexus birth palsy?
A.Oligohydramnios
B. Multigravida mother
C. Single birth cesarean delivery
D.Induced labor
A 10-year-old male sustained an angulated radial neck fracture.
Closed reduction was not successful, and the patient underwent
percutaneous reduction using a Kirschner wire (Figure 1). Injury to
the nearby nerve during this procedure is most likely to cause the
inability to
A. flex the thumb
interphalangeal (IP)
joint and index
finger distal
interphalangeal
(DIP) joint.
B. extend the wrist.
C. abduct and adduct
the fingers.
D. extend the thumb
IP joint.
Figures 1 and 2 are the radiographs of a 2-month-old male infant
who is unable to use his arm. The patient is afebrile and does not
appear to be ill. Physical examination of the elbow shows edema,
and attempts at passive range of motion elicit pain. Results of
laboratory studies are within normal limits. What is the best next
step in management?
A. Obtain an MRI of the
arm
B. Obtain a skeletal
survey and social work
consult
C. Apply a splint to the
elbow and schedule
follow-up
D. Aspirate the elbow
joint and send for
gram stain and culture
A 16-year-old male is transferred to the emergency department via
ambulance on a backboard and in a cervical collar after he is
involved in a rollover motor vehicle collision as a restrained
passenger. He says he has low back pain. Vital signs are within
normal limits. CT scans of the chest and abdomen are shown in
Figures 1 through 4. Which of the following commonly-associated
injuries is most likely to require urgent intervention in this patient?
A. Duodenum
B. Spinal cord
C. Liver
D. Iliac artery
Following a collision with another player, a 16-year-old female soccer
player has pain in the right knee and is brought to the emergency
department. Physical examination shows exquisite tenderness over the
anterior aspect of the knee with difficulty standing. The patient can
actively extend the knee to -10° of extension. No effusion or ligamentous
laxity is noted. Radiographs are shown in Figures 1 through 3. What is the
most appropriate treatment?
A. Open reduction and
internal fixation of the
fracture
B. Excision of the bone
fragment
C. Release of the lateral
retinaculum
D. Immobilization and
nonsteroidal anti-
inflammatory drugs
(NSAIDS)
A 13-year-old baseball pitcher who is left-hand dominant comes to
the clinic because he has had pain in his left shoulder for the past 3
weeks. His pitch counts have been <85 pitches on his pitching days
with 2+ days of rest between outings. Additionally, he has been
working with a pitching coach and practicing with his father who says
that he has a very good curve ball. Physical examination shows full
range of motion with tenderness to palpation over the proximal
humerus. Radiographs show a widened proximal humeral physis.
Repetition of what phase of the throwing motion is the most
common cause of the symptoms in this patient?
A. Stride
B. Late cocking
C. Acceleration
D. Follow through
Figure 1 is the radiograph of a 14-year-old female who has had
forefoot pain for the past 3 months. She is otherwise healthy. What
is the most appropriate initial treatment?
A. Metatarsal osteotomy
B. Excision of the fragment
C. Interposition arthroplasty
D. Immobilization in a cast or boot
Figures 1 and 2 are the radiographs of a 2-year-old female who is brought to the
emergency department because she has had pain and swelling in the right knee
since she fell 7 days ago. The parents say she had a bruise over her knee since the
fall, but this morning, she awoke with increased pain over the lower leg and was
limping. Temperature is 39.0°C (102.2°F), pulse rate is 180/min, respirations are
50/min, and blood pressure is 80/50 mmHg. The patient is very somnolent. On
physical examination, discomfort is noted with movement of the right leg. Edema
and warmth of the leg are noted. Laboratory studies show a white blood cell
count of 22,000/mm3 with 59% polymorphonuclear leukocytes and 25% bands;
erythrocyte sedimentation rate is 58 mm/hr; serum C-reactive protein level is 5
mg/L; and platelet count is 443,000/mm3. What is the most appropriate next step
in treatment?
A. Nonsteroidal anti-inflammatory drugs
and immobilization in a long leg cast
B. Outpatient MRI and analgesia
C. Emergent MRI and admission to the
ICU
D. Outpatient MRI and oral antibiotics
A 12-year-old female basketball player has had bilateral knee pain
for the past 2 years. She has had no formal treatment. Physical
examination shows a Beighton score of 6/9. Internal rotation of the
hip is to 90°, and external rotation of the hip is to 15°. No
ligamentous laxity is noted in the knee. Thigh-foot angle is to 30° of
external rotation. Examination of the gait shows a normal foot
progression angle with kissing patella. What is the most appropriate
next step in treatment?
A. CT of the knee
B. Physical therapy and nonsteroidal anti-inflammatory drugs
C. Femoral osteotomy
D. Femoral and tibial osteotomy
Figure 1 is the radiograph of a 4-year-old girl who is being evaluated
for genu varum. She has a family history of bowed legs and short
stature. She has a mutation in the PHEX gene. Identify the
laboratory studies most consistent with this diagnosi
A. Decreased phosphorus, increased serum
alkaline phosphatase, normal calcium and
vitamin D 25-OH
B. Decreased phosphorus and calcium, increased
serum alkaline phosphatase and increased
PTH, decreased 1,25 OH vitamin D
C. Increased phosphorus, increased calcium,
decreased alkaline phosphatase
D. Increased phosphorus, decreased calcium,
increased alkaline phosphatase, and increased
PTH
In the preoperative planning for the femoral lengthening of a patient
with the condition shown in Figure 1, evaluating which of the
following during the physical examination is critical in anticipating
postoperative complications?
A. Hip
B. Knee
C. Ankle
D. Gait
A 16-year-old elite ballerina has had pain in the left posterior ankle
for the past 6 months. She dances 5 to 6 hours a day, 6 days a
week. She says she has the most discomfort when she is en pointe.
The patient has no history of injury. Nonsteroidal anti-
inflammatory drugs and taping have not provided relief. Physical
examination shows pain over the posterior medial ankle with some
edema and pain with passive range of motion of the great toe. The
heel inverts with toe rise. Radiographs show an os trigonum but no
other abnormalities. Which of the following is the most likely
diagnosis?
A. Posterior tibial tendonitis
B. Flexor digitorum communis tendonitis
C. Plantar fasciitis
D. Posterior ankle impingement syndrome
A 2-year-old African American male who walked at 11 months of
age presents with bowed legs. His mother says that he is very active
and has no limitations, but he commonly trips and falls. Body mass
index is within normal limits for his age. On physical examination,
the thigh-foot angle is to 30° of internal rotation on the left and to
20° degrees on the right. Coverup test shows minimal genu varum
on the left but none on the right. Gait evaluation shows a moderate
lateral thrust on the left in the stance phase but not on the right.
Standing radiographs with the patellae forward show a
metaphyseal-diaphyseal angle of 11° without beaking. What part of
this patient's examination is most concerning for infantile Blount
disease?
A. Body mass index is 21kg/m2
B. Thigh-foot angle
C. Gait examination findings
D. Radiographic findings
A 9-month-old infant is being evaluated for nonaccidental
trauma in the emergency department. He is sent for a
skeletal survey. What fracture pattern has the highest
specificity for nonaccidental trauma?
A.Distal humerus physeal separation
B. Linear skull fracture
C. Proximal tibia corner fracture
D.Clavicle fracture
Figures 1 through 3 are the radiographs of a 12-year-old
female who sustained a twisting injury to her left ankle.
She has no history of pain prior to the injury, fever, or
systemic symptoms. Physical examination shows no skin
findings or soft tissue masses. A subsequent MRI scan is
shown in Figure 4, and results of a CT-guided biopsy are
shown in Figure 5. Based on these findings, what is the
most likely diagnosis?
A.Ollier disease
B. Osteosarcoma
C. Multiple hereditary exostosis
D.Jaffe-Campanacci syndrome
Although many subjective acute somatic concussion
symptoms are short-lived, less information is known
about the late effects of concussion and repetitive head
impact in patients. However, these patients are known to
be at increased risk for which of the following?
A.Anterior cruciate ligament injury
B. Anterior shoulder dislocation
C. Migraine headache
D.Amnesia
A 2-year-old male presents with a 4-day history of fever and leg pain that
progressed to an inability to bear weight. Laboratory studies show elevated white
blood cell count, serum C-reactive protein level, and erythrocyte sedimentation
rate. MRI scans show osteomyelitis of the distal femur with subperiosteal and
intraosseous abscesses. The patient undergoes irrigation and debridement
including a cortical window to decompress the intraosseous abscess. Cultures
grow methicillin-resistant Staphylococcus aureus (MRSA), and intravenous
antibiotics are initiated. The patient initially improves, and laboratory studies
show decreasing inflammatory markers. He is afebrile but continues to refuse to
bear weight. Three days later, evaluation shows fever, increasing serum C-reactive
protein level, and increased edema and erythema of the leg. Repeat MRI scans are
shown in Figures 1 and 2. The most appropriate next step in management is to
A. continue with intravenous antibiotics
and observation.
B. initiate anticoagulation for deep vein
thrombosis (DVT).
C. switch to a different antibiotic.
D. repeat irrigation and debridement.
Figures 1 through 3 are the radiographs of a 7-year-old male who is
being evaluated for pain in the right thigh and an episodic limp. He
has no history of injury. Body mass index is in the 99th percentile.
Physical examination shows a mild limp on the right with fairly
symmetric range of motion except for some mildly limited internal
rotation. What is the best next step in treatment?
A. In situ pinning of both
hips
B. Percutaneous pinning of
the right hip
C. Physical therapy and
follow-up in 6 weeks
D. In situ pinning of the right
hip with prophylactic
pinning of the left hip
A 3-year-old male is brought to the emergency department because he
has had a fever and has refused to bear weight during the past 2 days.
Current temperature is 39.0°C (102.2°F). Laboratory studies show a white
blood cell count of 13,000/mm3 with a left shift, serum C-reactive protein
level of 10 mg/L, and erythrocyte sedimentation rate of 50 mm/hr.
Results of blood cultures are pending. Plain radiographs show no
abnormalities. MRI scanning is unavailable for the next 2 days.
Ultrasonography of the hip is shown in Figure 1. What is the most
appropriate next step in treatment?
A. Initiate a trial of NSAIDs, and
discharge to home with strict return
precautions
B. Obtain a CT scan of the hip and pelvis
C. Admit patient, initiate antibiotic
therapy, and get the MRI scan in 2
days
D. Perform open irrigation and
debridement of the hip
Figures 1 and 2 are the clinical photograph and preoperative
radiograph of an 18-month-old male with syndromic scoliosis, and
Figure 3 is the intraoperative radiograph after application of a
Mehta cast. What is the primary benefit of treating this patient's
scoliosis with a Mehta cast?
A. There is no indication for
Mehta casting in this
diagnosis
B. Can delay the need for
future spinal surgery
C. Curative treatment for
the non-idiopathic
scoliosis
D. Decreases the incidence
of postoperative
pneumonia
Figures 1 and 2 are the standing radiograph and MRI scan of a 6-
year-old female with congenital femoral deficiency. On physical
examination, the pelvis levels with a 3-cm block under the short
leg. Lachman testing of the knee on the short side shows a soft
endpoint with a grade 2 laxity. If this patient undergoes lengthening
of the short leg, her risk of what unique complication is most likely
to increase?
A. Failure of distraction
B. Nonunion
C. Device breakage
D. Knee joint subluxation
Figures 1 through 4 are the radiographs and MR arthrogram images of a 12-year-
old male who is being evaluated for noticeably decreased range of motion of the
right elbow. The patient says that he started having right elbow pain almost a year
ago after he played in a dodgeball league with lots of side-arm throws. Physical
examination of the right elbow shows a significant lack of flexion (about 10°) and
lack of extension (approximately 30°) compared with the contralateral side. Range
of pronation and supination is full. No tenderness to palpation is noted at the
medial or lateral epicondyles, radial head, or olecranon. No varus-valgus instability
is noted. What is the most appropriate course of action?
A. Refer the patient to his pediatrician for
management
B. Initiate an aggressive physical therapy
and return-to-sport pathway
C. Immobilize in a long arm cast to allow
healing of the capitellar lesion
D. Order laboratory studies to determine
if there is an underlying systemic
disease
What is a strong predictive factor for femoral head
osteonecrosis that can be used as a screening tool in
patients with sickle cell disease (HbSS)?
A.Body mass index
B. Fetal hemoglobin
C. Glucocorticoid use
D.Ratio of hemoglobin to hematocrit
Figure 1 is the radiograph of a 14-year-old male who is right-hand
dominant and who fell while snowboarding. On physical
examination, the skin is intact, and neurovascular examination
shows no deficits. What is the most likely outcome of nonoperative
management of this injury?
A. Fracture union
B. Poor functional outcome
C. Faster return to activities
D. Symptomatic malunion
When compared with intravenous (IV) antibiotics via a
peripherally inserted central catheter, postdischarge
treatment of pediatric acute osteomyelitis with oral
antibiotics is associated with a
A.lower rate of treatment failure.
B. higher rate of treatment failure.
C. lower rate of rehospitalization or return visit to the
emergency department.
D.higher rate of rehospitalization or return visit to the
emergency department.
A 13-year-old hockey player sustains a concussion
during a play. Physical examination on the sideline
shows a loss of consciousness for more than 1 minute,
bloody otorrhea, and eyelid hematoma. What is the
most appropriate next step in management?
a) Emergency transportation to a hospital
b)Neuropsychologic testing to identify cognitive deficits
c) Return to home and follow-up with a neurologist
within 3 days
d)Return to home for physical rest and decreased
cognitive activity in the immediate 24 to 72 hours
Figures 1 and 2 are the radiographs of a 13-year-old male with a 6-
month history of left hip pain and a limp. Based on these findings,
an abnormality in which of the following serum values is most
likely?
A. Calcium
B. N-telopeptide
C. Luteinizing hormone
D. Leptin
Physical examination of a 2-week-old newborn shows
dislocation of the hip associated with developmental
dysplasia of the hip. Ultrasonography confirms the
clinical examination findings. What factor is most likely
to contribute to the successful outcome of Pavlik
harness treatment for this patient?
A.Female gender
B. Hip is dislocated at rest but reducible
C. Hip is classified as a Graf type IV on ultrasonography
D.A right sided dislocation
A 14-month-old toddler is diagnosed with early onset
scoliosis. Genetic workup is negative for any syndrome,
and radiographs do not show any congenital vertebral
malformation. What radiographic finding is most
predictive of spontaneous resolution of the curve?
A.Curve magnitude at presentation of >25
B. Phase 2 rib changes
C. Rib vertebral angle difference of <20
D.Presence of double curves
Figures 1 and 2 are the radiographs of a 4-year-old male
who is being evaluated for mild pain and swelling along
the left tibia as well as occasional limping. Histology of a
biopsy of the lesion is consistent with curvilinear
trabeculae of woven bone with osteoblastic rimming.
What is the most appropriate management?
A.Observation
B. Curettage and bone grafting
C. Wide extraperiosteal excision and reconstruction
D.Wide extraperiosteal excision and chemotherapy
A 17-year-old lineman sustained an injury to his right foot when
another player fell onto his heel while his foot was planted on the
ground. After the incident, the patient could not put pressure on his
right foot. Physical examination shows significant tenderness to
palpation of the midfoot. Plantar ecchymosis is noted. Non-weight
bearing radiographs show a subtle gap at the base of the first and
second metatarsals. MRI scans are most likely to show an injury to
the ligament between the base of the
A. first and second metatarsals.
B. first metatarsal and medial aspect of the middle cuneiform.
C. second metatarsal and lateral aspect of the medial cuneiform.
D. second metatarsal and medial aspect of the middle cuneiform.
A 5-year-old girl with X-linked dominant
hypophosphatemic rickets is evaluated in the clinic for
lower extremity deformities. Which of the following best
describes the pathology of the defective bone
mineralization with this disorder?
A.Poor intestinal phosphate absorption secondary to a
defect of 1 hydroxylation of 25-hydroxyvitamin D
B. Increased phosphate excretion through kidneys
secondary to increased fibroblast growth factor 23 level
C. Increased phosphate excretion through kidneys
secondary to proximal tubulopathy
D.Decreased levels of phosphate due to dysfunction of
alkaline phosphatase enzymes
A 12-year-old male is admitted to the hospital for
treatment of a stable slipped capital femoral epiphysis
on the left. What radiographic finding is most likely to
help the physician to determine if the contralateral hip is
at increased risk for subsequent slip?
A.Modified Oxford score ‚â•20
B. Superior epiphyseal extension ratio >0.71
C. Epiphyseal-diaphyseal angle difference >21°
D.Posterior epiphyseal extension angle >10° and open
triradiate cartilage
A 7-year-old male is evaluated in the emergency
department because he has an isolated midshaft fracture
of the tibia with an intact fibula. It is most appropriate for
the clinician to tell the patient's family that this fracture
pattern has a high tendency for what specific deformity?
A.Shortening
B. Varus
C. Malrotation
D.Recurvatum
Figures 1 and 2 are the radiographs of a premenarchal 12-year-old
female with adolescent idiopathic scoliosis. The Cobb angle of the
main thoracic curve measures 37°. A rigid thoracolumbosacral
orthosis is prescribed for treatment. What is the minimum number
of hours that this patient should wear the brace each day to
prevent surgery due to curve progression?
A. 18
B. 12
C. 10
D. 6
Figures 1 through 3 are the radiographs and MRI scan of the left tibia of a 15-
year-old male who comes to the emergency department because he has pain and
the inability to bear weight on the left side. His temperature is elevated.
Laboratory studies show a serum C-reactive protein level of 390 mg/L and an
erythrocyte sedimentation rate of 100 mm/hr. Blood cultures confirm a diagnosis
of methicillin-resistant Staphylococcus aureus (MRSA), and the patient undergoes
multiple surgical debridements to clear the infection. What factor is most likely
responsible for the bone and soft tissue toxicity seen during infections with this
offending organism?
A. Alkaline phosphatase
B. Matrix metalloproteinases
C. Panton-Valentine leukocidin
D. Parathyroid hormone-related protein
Following a difficult vaginal birth, a 2-week-old male is being evaluated for
Erb palsy. The mother is also concerned about swelling in the right leg and
left arm that has been present since birth. Figures 1 through 3 are the
radiographs of the right leg and left arm. The patient has not had any
associated fevers, and there is no suspicion of nonaccidental trauma since
discharge from the hospital. Laboratory studies show elevation of
erythrocyte sedimentation rate, serum alkaline phosphatase level, and
serum C-reactive protein level. Which of the following other locations in
the body most frequently displays the same radiographic findings and is
pathognomonic for this diagnosis?
A. Metatarsals
B. Iliac wing
C. Vertebra
D. Mandible
A 9-year-old male presents with a history of fevers and a limp.
Current temperature is 38.9°C (102.0°F). Laboratory studies show an
erythrocyte sedimentation rate of 100 mm/hr, white blood cell
count of 18,200/mm3, and serum C-reactive protein level of 393
mg/L. An MRI scan shows a large subperiosteal abscess around the
right tibia and a large effusion in the ankle. The patient immediately
undergoes irrigation and debridement of both areas, which is
repeated several times over the next few days. Eventually, the
patient is discharged to home. Two weeks later, the patient has
development of a venous thromboembolism (VTE) that is treated
with anticoagulation. Which data point in this patient's workup is
the strongest predictor of the development of a VTE?
A. White blood
B. C-reactive protein
C. Erythrocyte sedimentation rate
D. Temperature
A 7-year-old female is brought to the emergency department
because she has a temperature to 38.8°C (101.8°F), has pain in the
right knee, and refuses to walk or crawl. Physical examination shows
no tenderness with moderate passive range of motion of the knee.
Laboratory studies show an erythrocyte sedimentation rate of 120
mm/hr and a serum C-reactive protein level of 250 mg/L. Aspiration
of the right knee yields cloudy fluid with a cell count of
60,000/mm3. When comparing the application of the Kocher
criteria to a case of presumed septic arthritis of the knee to the
application of the criteria to a case of septic arthritis of the hip, the
Kocher criteria
A. yields a decreased sensitivity.
B. are more sensitive when the WBC count is < 8,000/mm3.
C. is less sensitive when C-reactive protein values are added.
D. is indistinguishable.
A 4-year-old girl who has midlumbar myelomeningocele and
hydrocephalus is being evaluated for her uneven gait. A hip
examination reveals a right hip dislocation. Bilateral hips are pain-
free with full range of motion and no fixed deformity. What is the
most appropriate course of action?
A. Obtain standing AP pelvis to evaluate the hips; plan open
reduction of the right hip when the patient starts having hip pain
B. Obtain supine AP pelvis to evaluate the hips; plan open reduction
of the right hip as soon as possible
C. Inform the family that the dislocated hip will not limit the ability
to ambulate and will not benefit from reduction
D. Inform the family that the patient's ambulatory potential is poor
due to the hip dislocation, which cannot be treated successfully
What clinical finding is more likely in pediatric patients
with methicillin-resistant Staphylococcus aureus (MRSA)
osteomyelitis than methicillin-sensitive Staphylococcus
aureus (MSSA) osteomyelitis?
A.Concurrent septic arthritis
B. Deep vein thrombosis
C. Multifocal osteomyelitis
D.Subsequent endocarditis
A 16-year-old football player is being evaluated because he
has groin pain with internal rotation and flexion of the hip.
An MRI scan of the hip shows an acetabular labral tear,
and radiographs show an alpha angle of 75°. When
counseling the patient and his family about next steps,
what is the most appropriate recommendation?
A.Arthroscopic labral repair
B. Arthroscopic femoroplasty
C. Surgical hip dislocation with labral repair
D.Physical therapy and anti-inflammatory medications
A 14-year-old female sustains an anterior cruciate
ligament (ACL) injury while participating in a soccer
game. Menstruation began at 12 years of age. When
counseling this patient and her family about treatment
options, what option has the highest chance of success
with good patient outcomes?
A.Extra-articular reconstruction with the iliotibial band
(ITB)
B. Anatomic ACL reconstruction with allograft tissue
C. Nonoperative bracing until 16 years of age and then
ACL reconstruction
D.Anatomic ACL reconstruction with autograft tissue
An 11-month-old female is evaluated in the emergency
department because she has a midshaft fracture of the
femur. The patient has met all developmental milestones
but has not yet ambulated. What other findings on
history or physical examination are most likely to suggest
nonaccidental trauma as the cause of this patient's
fracture?
A.Bruises and cutaneous markings
B. Female sex
C. Parents serving in the military
D.Paternal grandmother living in the home
Figure 1 is the radiograph of a 12-year-old baseball pitcher who has a 3-
month history of vague shoulder pain and no history of injury. He says he
has no pain at rest but feels lateral-based pain after 10 to 15 throws.
Physical examination shows decreased internal rotation with the arm in
abduction compared with the contralateral side and 5/5 strength in the
rotator cuff with no pain and no apprehension. Speed test elicits some
mild discomfort. What is the most appropriate next step in management?
A. MRI arthrography to rule out labral
pathology
B. Switch positions to catcher for the
upcoming season
C. Complete rest from throwing for 6
weeks and then a graduated return
to throwing
D. Immediate physical therapy consult
to work on throwing mechanics
and rotator cuff strengthening
A fully immunized 16-month-old female is being evaluated because
she has irritability and pain in the right hip. She refuses to bear weight
on the leg. Temperature is 37.9°C (100.3°F). On physical examination,
the hip is held in flexion and external rotation, and pain is elicited
with rotational movements. Laboratory studies show a white blood
cell count of 10,900/mm3, erythrocyte sedimentation rate of 18
mm/hr, and serum C-reactive protein level of 16 mg/L.
Ultrasonography shows a small effusion, and analysis of synovial fluid
shows white blood cells of 80,000/mm3 with 88% neutrophils.
Immediate irrigation and debridement is performed, but cultures
obtained previously and intraoperatively do not grow a definitive
organism. What is the most likely diagnosis?
A. Septic arthritis with methicillin-resistant Staphylococcus aureus
B. Transient synovitis
C. Septic arthritis from Haemophilus influenzae type b
D. Septic arthritis from Kingella kingae
A 1-month-old female is being evaluated because she has
not been moving her right arm. Physical examination
shows asymmetric pupils, and the right pupil is
constricted. No movement of the right upper extremity
occurs when the neonatal reflexes are stimulated. It is
most appropriate to tell the mother that the outcome of
this patient's condition is most likely to be poor because
of which of the following reasons?
A.There is evidence that the sympathetic chain is involved
B. There has not yet been a return of biceps function
C. There is evidence of postganglionic involvement
D.There is evidence of Erb palsy
A 16-year-old high school football player sustains a first-
time anterior glenohumeral dislocation during the second
game of the season. The patient and his family would like
to pursue conservative management with return to play
during the same season. Imaging studies do not show a
glenoid fracture or large bony Bankart lesion. It is most
appropriate to tell the patient and his family that
A.bracing of the shoulder has demonstrated lower rates
of repeat instability events.
B. the risk of recurrent instability in this patient is high.
C. the patient will likely be able to return to play in 1 to 2
weeks.
D.returning to play during the same interscholastic season
is not an option.
In a 3-year-old patient, the anterior humeral line crosses
the anterior third of the capitellar ossific nucleus with
what frequency when compared with the middle third?
A.Similar
B. Decreased
C. Increased
D.Variable based on hand dominance
A 2-week-old male is brought to the office by his mother because
he has had fevers and irritability for the past 24 hours. The patient
was born at full term via vaginal delivery with no perinatal
complications. Physical examination shows that the right hip is
held in a flexed position. The patient cries with attempts to extend
the hip. Laboratory studies show a white blood cell
A. Repeat ultrasonography
of the hip in 1 week
B. Place the patient in a
Pavlik harness
C. Administer antibiotic
therapy
D. Aspirate the right hip
count of 12,400/mm3, erythrocyte sedimentation rate of 44 mm/hr,
and serum C-reactive protein level of 76 mg/L. Ultrasonography is
shown in Figure 1. What is the most appropriate next step?
A. Arthroscopic reconstruction of
the anterior cruciate ligament
(ACL)
B. Physical therapy for range of
motion and extension stretches
C. Arthroscopic evaluation and
internal fixation of the disp
Figures 1 through 4 are the radiograph and MRI scans of a 12-year-
old female soccer player who has had a large effusion and pain in the
left knee after she sustained a noncontact injury 2 days ago. Physical
examination shows limited painful motion, diffuse tenderness to
palpation medially and laterally, and guarding during Lachman
testing. What is the most appropriate next step in management?
A. Earlier time to union of the
fracture
B. Delayed time to union of the
fracture
C. Greater functional improvement
at 6 weeks
D. Higher malunion rate of the
fracture
Figures 1 and 2 are the radiographs of a 10-year-old female who
tripped while running. A long leg cast is applied, and the patient is
encouraged to begin weight bearing as tolerated. What outcome can
the family expect to see compared with non-weight bearing in the
cast?
A. Perform hip capsulotomy and remeasure epiphyseal perfusion
B. Observe the patient for appropriate healing and monitor for
avascular necrosis (AVN)
C. Perform open femoroplasty to prevent secondary impingement
D. Remove the screw and reposition the femoral head to improve
perfusion
Figure 1 is the radiograph of a 10-year-old gymnast who has had left
wrist pain for the past 2 months. Physical examination shows no
edema, mild tenderness at the distal radial physis, and pain with
forced dorsiflexion of the wrist. What is the most appropriate next
step in management?
A. Casting of the wrist for 6 weeks
B. Distal ulnar epiphysiodesis
C. Surgical stabilization with
smooth pins
D. Activity modification and
metabolic workup
The four anatomic sites with highest risk of concomitant
pediatric septic arthritis and osteomyelitis are
A.proximal humerus, distal femur, proximal femur, and
distal forearm.
B. proximal humerus, proximal ulna, proximal femur, and
proximal tibia.
C. proximal humerus, proximal radius, proximal femur, and
distal fibula.
D.proximal radius, distal forearm, proximal femur, and
distal tibia.
Figure 1 is the radiograph of a 12-year-old male who has activity
related right knee pain and a painful click with activity. Physical
examination shows a small effusion, stable results on Lachman
testing, and no patellar apprehension. Pain is elicited with internal
tibial rotation moving from flexion to extension. What is the best
next step in management?
A. MRI of the knee to evaluate lesion
stability
B. Triple-phase bone scan to evaluate
lesion activity
C. CT scan to further determine the
extent of the lesion
D. Immobilization in a long leg cast for 6
weeks then repeat evaluation
Figures 1 and 2 are the radiographs of a 10-year-old soccer player
who has a 2-day history of limping with the left leg as well as pain in
the left ankle after she sustained an inversion injury. Physical
examination shows mild edema of the lateral ankle with tenderness
to palpation along the distal fibula. No medial tenderness is noted.
Results of anterior drawer and talar tilt tests show stability. The best
next step in management is
A. excision of the symptomatic os
subfibulare with the anterior
talofibular ligament.
B. open reduction and fixation of
the epiphyseal avulsion
fracture.
C. use of a CAM walker boot and
weight bearing as tolerated for
3 to 4 weeks.
D. MRI to assess ligamentous
injury.
Figure 1 is the radiograph of a 14-month-old female who is brought
to the emergency department because she refuses to bear weight
on her leg. What factor is correlated with nonaccidental trauma
(NAT) in this injury?
A. Patient is ambulatory
B. Fracture pattern is comminuted
C. Patient age is <2 years
D. Health care coverage is by
commercial insurance
A youth baseball player has recurrence of proximal
humeral epiphysiolysis. Physical examination of shoulder
motion in this patient is most likely to show
A.increased shoulder external rotation.
B. decreased shoulder external rotation.
C. increased shoulder rotational arc of motion.
D.decreased shoulder rotational arc of motion.
An 8-year-old male is being treated for acute
hematogenous osteomyelitis of the proximal tibia with
empiric intravenous antibiotic therapy. After 48 hours, no
clinical improvement is noted, and the patient remains
febrile. What is the best next step?
A.Obtain an MRI of the knee and tibia
B. Obtain an ultrasonography of the knee
C. Change to a more broad-spe development. Therefore,
screening abdominal ultrasonography followed by serial
ultrasounds every 3 to 4 months until age 7 to monitor
for tumor development is recommended.
A 5-year-old female with Beckwith-Wiedemann syndrome is being
evaluated for unequal leg lengths. Physical examination and
radiographs show a 1-cm leg length discrepancy and increased
girth of the left leg. What diagnostic study is the most appropriate
next step?
A. Abdominal ultrasonography
B. MRI of the lower extremities
C. Hip ultrasonography
D. MRI of the brain
In children with Erb palsy, shoulder contracture and joint
deformity develop secondary to weak
A.external rotators and abductors.
B. internal rotators and abductors.
C. external rotators and adductors.
D.internal rotators and adductors.
Figures 1 and 2 are the radiographs of an 11-year-old male
who fell off a trampoline and sustained a Salter-Harris
type II fracture of the distal femur. Which parameter is an
independent risk factor for the need of additional
surgeries to treat associated leg length or angular
deformities of the knee that may subsequently develop?
A.Type of instrumentation
used
B. Sex of patient
C. Age of patient
D.Ossification of the tibial
tuberosity
Figure 1 is the lateral radiograph of the right calcaneus of an 11-
year-old male soccer player who has had pain in the heel for the
past 6 weeks that has limited his ability to play. He plays on 2 travel
teams, and 1 day a week the patient works with a fitness coach who
thinks he needs more plyometric training. On physical examination,
the skin is intact. No evidence of neurovascular compromise is
noted. Pain is elicited when the calcaneus is squeezed. What is the
most appropriate next step in treatment?
A. CT to rule out a tarsal coalition
B. Injection of the calcaneal apophysis
with platelet rich plasma
C. Rest, ice, use of heel pads, physical
therapy, and nonsteroidal anti-
inflammatory drugs
D. Drilling the calcaneal apophysis at 4
sites with a smooth 2-mm Kirschner
wire
A 12-year-old female who is 3 months post menarche is being
evaluated for spinal curvature. A detailed neurological examination
shows no abnormalities. Radiographs of the spine (Figures 1 and 2)
show an apex left lumbar curve measuring 36° and an apex right
thoracic curve measuring 33°. What is the most appropriate
treatment?
A. Thoracolumbosacr
al orthosis
B. Posterior spinal
fusion
C. Growing rods
D. Observation
A 6-year-old male sustains a closed midshaft tibia and fibula
fracture after falling off of a trampoline. The patient undergoes
closed reduction in the emergency department, and post-reduction
alignment is acceptable. At follow-up 1 week later, the fracture has
maintained reduction with immobilization in a long leg cast. The
parents want to know when their son can start walking on the cast.
Based on the literature, early weight bearing for pediatric tibia and
fibula fractures is most likely to result in
A. earlier return to activity.
B. higher rates of malunion at final follow-up.
C. fracture displacement and higher rates of surgical intervention.
D. decreased fracture healing time with no increase in
complications.
Figure 1 is the radiograph of a 9-year-old male who has persistent
pain in the left upper arm with activity. Medical history includes a
fracture of the proximal humerus 1 year ago. What is the most likely
diagnosis?
A. Aneurysmal bone cyst
B. Telangiectatic osteosarcoma
C. Little league shoulder
D. Unicameral bone cyst
A 13-year-old competitive female gymnast has had bilateral elbow
pain for the past 6 months. She says she feels clicking and popping
in the elbow. Radiographs are shown in Figures 1 and 2, and Figure
3 is an MRI scan that shows loose cartilage bodies in the joint. What
is the most appropriate treatment?
A. Cessation of gymnastics activity
B. Elbow arthroscopy with loose body removal and associated
microdrilling
C. Open reduction and internal fixation of the osteochondral
fragment
D. Physical therapy to work on elbow and shoulder range of
motion and strengthening exercises
A 12-year-old male pitcher who is right-hand dominant is
being evaluated for pain in the anterior lateral shoulder
that is associated with pitching. The pain is severe in the
late cocking phase. The patient pitches on 2 separate
baseball teams year round and has a private pitching
instructor. What finding is most likely on MRI scan of this
patient?
A. Labral and rotator cuff pathology
B. Proximal humeral physeal narrowing
C. Metaphyseal edema adjacent to the physis
D. Stress fractures in the proximal humeral shaft
Figure 1 is the radiograph of a 13-year-old male basketball player
who has a 3-month history of throbbing pain over the anterior knee
with activity. He plays basketball on his school team and on a
competitive club team. He also runs track to improve his vertical
jump. Physical examination shows tenderness to palpation at the
tibial tubercle. What is the most appropriate treatment?
A. Rest, ice, nonsteroidal anti-
inflammatory drugs, and
stretching
B. Surgical excision of the
fragmented apophysis ossicles
C. Injection of platelet rich plasma
D. MRI of the knee to evaluate for
interarticular pathology
An 8-year-old male is admitted to the hospital for treatment of
distal femur osteomyelitis with a subperiosteal abscess. Culture of
the abscess is positive for methicillin-resistant Staphylococcus
aureus. What role does coagulase play in this patient's
musculoskeletal infection?
A. Adherence of Staphylococcus aureus to fibrin protecting it from
phagocytosis
B. Activation of prothrombin to thrombin leading to clot and
abscess formation
C. Lysis of neutrophils due to pore formation at the cell surface
D. Activation of platelets and bacterial adhesion to blood vessel
walls
An otherwise healthy 2-year-old female is brought to the emergency
department because she has refused to bear weight on the right leg
for the past 2 days. The patient has no history of trauma, and she
attends day care. Temperature is 38.0°C (100.4°F). On physical
examination, pain is elicited on movement of the right hip. On
laboratory studies, serum C-reactive protein level is 16 mg/L, and
white blood cell count and erythrocyte sedimentation rate are within
normal limits. An MRI scan of the right hip shows proximal femoral
osteomyelitis with surrounding joint effusion. The patient undergoes
irrigation and debridement of the right hip and proximal femur. In
addition to aerobic bacterial cultures, what other test is most
appropriate to identify the most likely causative bacteria?
A. Fungal culture
B. Acid-fast bacteria (AFB)
C. Anaerobic bacterial culture
D. Polymerase chain reaction (PCR)
A 13-year-old male with adolescent Blount disease is undergoing
hemiepiphysiodesis with a tension band to correct the mechanical
axis. Body mass index is 62 kg/m2. The medial proximal tibial angle
for the affected limb is 70°. Which implant is most likely to provide
the lowest risk of failure?
A. 1 epiphyseal and 1 metaphyseal cannulated titanium screw
B. 2 epiphyseal and 2 metaphyseal solid titanium screws
C. 1 epiphyseal and 1 metaphyseal cannulated stainless steel
screw
D. 2 epiphyseal and 2 metaphyseal solid stainless steel screws
A 14-year-old female who is skeletally mature is brought to the emergency
department after she sustained a high-energy open tibia fracture in a snowmobile
accident. Tertiary survey is negative for any additional trauma. Intraoperative
findings show compartments that are soft and swollen but compressible. Distal
vascularity is intact, and wound edges cannot be reapproximated. The clinical
photograph in Figure 1 shows the appearance of a 17-cm open wound following
emergent excisional irrigation and debridement and initiation of intravenous
antibiotics. In addition to plastic surgical flap coverage in 24 to 48 hours, what is
the most appropriate immediate surgical treatment?
A. Loose wound reapproximation, sterile dressing
application, use of a long leg posterior splint, plan to
repeat irrigation and debridement, and intramedullary
nailing
B. Application of a ring external fixator, vacuum-assisted
dressing application, and plan to repeat irrigation and
debridement
C. Immediate intramedullary nailing, vacuum-assisted
dressing application, and plan to repeat irrigation and
debridement
D. Immediate submuscular plate osteosynthesis, vacuum-
assisted dressing application, and plan to repeat
irrigation and debridement
Figures 1 and 2 are the radiographs of a 6-year-old male who is
brought to the emergency department after he sustained an isolated
femoral shaft fracture after being struck by a car while riding a
scooter. What is the most appropriate treatment?
A. Submuscular plating
B. Immediate hip spica casting
C. Flexible intramedullary nailing
D. Locked intramedullary nailing
with trochanteric entry
Figures 1 and 2 are the sagittal CT scan and sagittal STIR sequence MRI scan of a
15-year-old male who is brought to the emergency department after he sustained
an injury to his spine in a high-speed motor vehicle collision. He was a restrained
rear-seat passenger and was wearing a lap belt. Physical examination shows
hypovolemic shock, a distended abdomen, ecchymosis across the anterior
abdomen, and bilateral weakness in quadriceps and ankle dorsiflexion. What is
the most appropriate surgical treatment?
A. Immediate open reduction,
decompression, and fusion of the
compression fracture of the lumbar spine
B. General surgical exploratory laparotomy
followed by staged treatment of the
compression fracture of the lumbar spine
with an extension thoracolumbosacral
orthosis
C. General surgical exploratory laparotomy
followed by staged instrumented
reduction and fusion of the Chance
fracture of the lumbar spine
D. Immediate instrumented reduction and
fusion of the Chance fracture of the
lumbar spine followed by general surgical
exploratory laparotomy
Figure 1 is the radiograph of a 6-year-old male with Down syndrome
whose parent says that they can hear a popping in his right hip.
Physical examination shows a Trendelenburg gait and a leg length
discrepancy. Barlow test is positive in the right hip. What is the most
appropriate treatment?
A. Adductor longus tenotomy, arthrography, closed reduction, and
application of a hip spica cast
B. Open reduction and capsulorrhaphy, acetabular osteotomy, and
proximal femoral varus osteotomy
C. Proximal femoral varus osteotomy only
D. Closed reduction and spica casting
A 5-year-old patient presents to the emergency department with a
right knee effusion, antalgic gait, and limited range of motion of the
knee. The patient is afebrile. Physical examination shows moderate
limitation of passive range of motion of the knee from 20 to 90° of
flexion, and pain is noted at the end points. Laboratory studies show
a white blood cell count (WBC) of 13,500/mm3 and erythrocyte
sedimentation rate (ESR) of 42 mm/hr. What is the most appropriate
next step in treatment?
A. Knee aspiration and synovial fluid analysis
B. Urgent arthroscopic surgical lavage of the knee
C. Admission to the hospital for observation with initiation of
intravenous antibiotics
D. Discharge to home with oral nonsteroidal anti-inflammatory
drugs
Figures 1 and 2 are the coronal and sagittal STIR MRI scans of a 7-year-old
female who has had worsening pain in the left ankle over the past week and
new onset of inability to bear weight on the leg upon awaking this morning.
Temperature is 39.0°C (102.2°F). Laboratory studies show a white blood cell
count of 15,500/mm3, serum C-reactive protein level of 16.7 mg/L, and
erythrocyte sedimentation rate of 50 mm/hr. What is the best next step?
A. Arthroscopic lavage of the left ankle
B. Surgical irrigation and debridement of the left distal tibia
C. CT scan of the chest and an oncology consult
D. Empiric intravenous antibiotics and
E. serial inflammatory markers to assess response
Figure 1 is the radiograph of a 13-year-old basketball player who
twisted her ankle during a layup. She says she felt a pop and limped
off the court. On physical examination, the skin is intact.
Neurovascular examination shows no abnormalities. This patient's
injury is most likely caused by avulsion of what ligament?
A. Anterior-inferior tibiofibular
B. Calcaneofibular
C. Anterior talofibular
D. Plantar facial ligament
A Tillaux fracture occurs when the anterior-inferior tibiofibular
ligament avulses an epiphyseal fragment off the anterolateral tibia,
typically from an external rotation mechanism. A Salter-Harris type
III fracture occurs. This is a common adolescent sports injury. Ankle
physeal closure occurs between 12 to 15 years of age. The distal
tibia physis closes from medial to lateral, leaving the lateral physis
vulnerable to this transitional avulsion fracture. The radiograph
shows the typical appearance of this fracture. The anterior
talofibular ligament, posterior-inferior tibiofibular ligament, and
calcaneofibular ligament are not involved.
A. Glascow coma scale.
B. Sport Concussion Assessment Tool 5 (SCAT5).
C. EyeBoxCNS test.
D. Immediate Post-Concussion Assessment and Cognitive Testing
(ImPACT) battery.
A 12-year-old female who is in eighth grade has had progressive
bowing of the left leg over the past 2 years. She has no history of
trauma to the leg, and she does not have pain. Weight is 249 lb. On
physical examination, the right leg is straight. Radiographs of the
left knee are shown in Figures 1 and 2. Adolescents with this
disorder also most commonly have what medical condition?
A. Vitamin D deficiency
B. Amenorrhea
C. Hypertension
Figures 1 through 4 are the radiographs and MRI scans of a 16-year-old
male lacrosse player who has had pain and swelling of the left knee for the
past 3 weeks. He has not had fever, weight loss, or change in appetite, and
he has no history of the knee giving way. He is afebrile. Laboratory studies
show a white blood cell count of 9000/mm3, erythrocyte sedimentation
rate of 32 mm/hr, and serum C-reactive protein level of 0.8 mg/L. What is
the most likely diagnosis?
A. Anterior cruciate ligament (ACL)
tear
B. Osteosarcoma
C. Ewing sarcoma
D. Chondroblastoma
Figures 1 and 2 are the radiographs of a 15-year-old male who comes
to the emergency department because he has had pain and swelling
of the right knee since he landed a jump shot during a basketball
game. It is most appropriate to also assess this patient for what
associated injury?
A. Posterior cruciate
ligament tear
B. Compartment
syndrome
C. Common peroneal
injury
D. Fibular fracture
An 18-month-old male is being evaluated for a right flatfoot deformity.
Physical examination shows a rigid deformity of the foot. Radiographs of
the foot are shown in Figures 1 and 2. What is the most appropriate
treatment?
A. Observation
B. Bracing with a supramalleolar
orthosis
C. Serial casting followed by surgical
correction
D. Medial column shortening with
lateral column lengthening
A 16-year-old-high school wrestler is referred by his
primary care physician for low back pain that has been
present for the past 3 months. Physical examination shows
pain with extension of the lumbar spine. The patient
cannot touch the toes when asked to bend forward.
Neurologic examination shows no focal deficits, and result
of straight leg raise test is negative. What is the most
appropriate initial imaging study of the lumbar spine?
A.Anteroposterior and lateral radiographs
B. Anteroposterior, lateral, and oblique radiographs
C. CT scan
D.SPECT scan
Of the following genes that are necessary for normal
hindfoot development, which has been implicated as a
cause of clubfoot?
A. PITX1-TBX4
B. HOX
C. FLNBM
D. TGFB1
A very large 1-week-old female infant presents to the orthopaedic
clinic after undergoing a difficult delivery. The parents are
concerned that the patient is not moving her right upper extremity
as much as the left. On physical examination, the right upper
extremity is resting in adduction and internal rotation with the wrist
in flexion. Full passive range of motion is noted at the shoulder,
elbow, wrist, and digits. The most appropriate treatment is
A. immediate brachial plexus nerve transfers.
B. a daily regimen of passive range of motion exercises and
stretching.
C. planned brachial plexus nerve transfers when the patient is 6 to
12 months old.
D. planned humeral derotational osteotomy when the patient is 18
months old.
Figures 1 through 3 are the clinical photographs of a 4-year-old
female who has an abnormal gait. She underwent serial casting as
an infant and used boots-and-bar orthotics for about 1 year
afterward. The recurrent deformity is passively correctable. What is
the most appropriate next step in treatment?
A. Use of a boots-and-bar orthosis and refer to physical therapy for
gait retraining
B. Perform circumferential posteromedial release followed by use of
a boots-and-bar orthosis
C. Repeat serial casting followed by tibialis anterior tendon transfer
D. Repeat serial casting and refer to physical therapy for gait
retraining
Figure 1 is the radiograph of a 15-year-old male who presents to the
orthopaedic office because he has a several-month history of low back
pain. He is active in football, wrestling, and weight lifting, and he has no
history of trauma or injury. He says the pain is aggravated by prolonged
sitting and is relieved by movement and taking nonsteroidal anti-
inflammatory drugs. Physical examination shows limited forward flexion.
The patient is able to heel-to-toe walk without difficulty. Straight leg raise
test does not elicit significant pain. What is the most likely diagnosis?
A. Disk herniation
B. Spinal cord tumor
C. Scheuermann disease
D. Thoracolumbar fracture

More Related Content

Similar to AAOS 2022 Pediatric Questions Answers and Discussion.pptx

Final Draft Case Report
Final Draft Case ReportFinal Draft Case Report
Final Draft Case Report
David Coyne
 
Pentagon Intraarticular Osteotomy: A Novel Surgical Approach to Complex Defor...
Pentagon Intraarticular Osteotomy: A Novel Surgical Approach to Complex Defor...Pentagon Intraarticular Osteotomy: A Novel Surgical Approach to Complex Defor...
Pentagon Intraarticular Osteotomy: A Novel Surgical Approach to Complex Defor...
skisnfeet
 
5. PCL repair
5. PCL repair5. PCL repair
5. PCL repair
drajun
 
Low back injury in a football lineman
Low back injury in a football linemanLow back injury in a football lineman
Low back injury in a football lineman
Jayson Nielsen
 
Open debridement and radiocapitellar replacement in primary and post-traumati...
Open debridement and radiocapitellar replacement in primary and post-traumati...Open debridement and radiocapitellar replacement in primary and post-traumati...
Open debridement and radiocapitellar replacement in primary and post-traumati...
Alberto Mantovani
 

Similar to AAOS 2022 Pediatric Questions Answers and Discussion.pptx (18)

Final Draft Case Report
Final Draft Case ReportFinal Draft Case Report
Final Draft Case Report
 
Dunaway a
Dunaway aDunaway a
Dunaway a
 
Use_of_zoledronic_acid_in_pelvic_and_sacral.2.pdf
Use_of_zoledronic_acid_in_pelvic_and_sacral.2.pdfUse_of_zoledronic_acid_in_pelvic_and_sacral.2.pdf
Use_of_zoledronic_acid_in_pelvic_and_sacral.2.pdf
 
case discussion 4
case discussion 4case discussion 4
case discussion 4
 
Pentagon Intraarticular Osteotomy: A Novel Surgical Approach to Complex Defor...
Pentagon Intraarticular Osteotomy: A Novel Surgical Approach to Complex Defor...Pentagon Intraarticular Osteotomy: A Novel Surgical Approach to Complex Defor...
Pentagon Intraarticular Osteotomy: A Novel Surgical Approach to Complex Defor...
 
Stemcell Research Paper on avascular necrosis-AVN-by Dr.Pradeep Mahajan
Stemcell Research Paper on avascular necrosis-AVN-by Dr.Pradeep MahajanStemcell Research Paper on avascular necrosis-AVN-by Dr.Pradeep Mahajan
Stemcell Research Paper on avascular necrosis-AVN-by Dr.Pradeep Mahajan
 
czmolekcasestudy
czmolekcasestudyczmolekcasestudy
czmolekcasestudy
 
5. PCL repair
5. PCL repair5. PCL repair
5. PCL repair
 
Anterior cruciate injuries and management (2).pptx
Anterior cruciate injuries and management (2).pptxAnterior cruciate injuries and management (2).pptx
Anterior cruciate injuries and management (2).pptx
 
Low back injury in a football lineman
Low back injury in a football linemanLow back injury in a football lineman
Low back injury in a football lineman
 
Poster Tl Sciatic Scoliosis
Poster Tl Sciatic ScoliosisPoster Tl Sciatic Scoliosis
Poster Tl Sciatic Scoliosis
 
Escoliosis
EscoliosisEscoliosis
Escoliosis
 
Ankylosing spondylitis.pptx
Ankylosing spondylitis.pptxAnkylosing spondylitis.pptx
Ankylosing spondylitis.pptx
 
Medial epicondyle fracture-pediatric.ppt
Medial epicondyle fracture-pediatric.pptMedial epicondyle fracture-pediatric.ppt
Medial epicondyle fracture-pediatric.ppt
 
Open debridement and radiocapitellar replacement in primary and post-traumati...
Open debridement and radiocapitellar replacement in primary and post-traumati...Open debridement and radiocapitellar replacement in primary and post-traumati...
Open debridement and radiocapitellar replacement in primary and post-traumati...
 
Spine Lecture Intraoperative neuromonitoring changes June 2019
Spine Lecture Intraoperative neuromonitoring changes June 2019Spine Lecture Intraoperative neuromonitoring changes June 2019
Spine Lecture Intraoperative neuromonitoring changes June 2019
 
Conference ext.สิทธิกร ปภาวิน orthokorat 1
Conference ext.สิทธิกร ปภาวิน orthokorat 1Conference ext.สิทธิกร ปภาวิน orthokorat 1
Conference ext.สิทธิกร ปภาวิน orthokorat 1
 
Exploring Advances In THA
Exploring  Advances In  THAExploring  Advances In  THA
Exploring Advances In THA
 

Recently uploaded

Cytoskeleton and Cell Inclusions - Dr Muhammad Ali Rabbani - Medicose Academics
Cytoskeleton and Cell Inclusions - Dr Muhammad Ali Rabbani - Medicose AcademicsCytoskeleton and Cell Inclusions - Dr Muhammad Ali Rabbani - Medicose Academics
Cytoskeleton and Cell Inclusions - Dr Muhammad Ali Rabbani - Medicose Academics
MedicoseAcademics
 
Failure to thrive in neonates and infants + pediatric case.pptx
Failure to thrive in neonates and infants  + pediatric case.pptxFailure to thrive in neonates and infants  + pediatric case.pptx
Failure to thrive in neonates and infants + pediatric case.pptx
claviclebrown44
 
CAD CAM DENTURES IN PROSTHODONTICS : Dental advancements
CAD CAM DENTURES IN PROSTHODONTICS : Dental advancementsCAD CAM DENTURES IN PROSTHODONTICS : Dental advancements
CAD CAM DENTURES IN PROSTHODONTICS : Dental advancements
Naveen Gokul Dr
 

Recently uploaded (20)

Lachesis Mutus- a Homoeopathic medicinel.pptx
Lachesis Mutus- a Homoeopathic medicinel.pptxLachesis Mutus- a Homoeopathic medicinel.pptx
Lachesis Mutus- a Homoeopathic medicinel.pptx
 
parliaments-for-health-security_RecordOfAchievement.pdf
parliaments-for-health-security_RecordOfAchievement.pdfparliaments-for-health-security_RecordOfAchievement.pdf
parliaments-for-health-security_RecordOfAchievement.pdf
 
Signs It’s Time for Physiotherapy Sessions Prioritizing Wellness
Signs It’s Time for Physiotherapy Sessions Prioritizing WellnessSigns It’s Time for Physiotherapy Sessions Prioritizing Wellness
Signs It’s Time for Physiotherapy Sessions Prioritizing Wellness
 
Cytoskeleton and Cell Inclusions - Dr Muhammad Ali Rabbani - Medicose Academics
Cytoskeleton and Cell Inclusions - Dr Muhammad Ali Rabbani - Medicose AcademicsCytoskeleton and Cell Inclusions - Dr Muhammad Ali Rabbani - Medicose Academics
Cytoskeleton and Cell Inclusions - Dr Muhammad Ali Rabbani - Medicose Academics
 
Hi Fi * Surat ℂall Girls Surat Dumas Road 8527049040 WhatsApp AnyTime Best Su...
Hi Fi * Surat ℂall Girls Surat Dumas Road 8527049040 WhatsApp AnyTime Best Su...Hi Fi * Surat ℂall Girls Surat Dumas Road 8527049040 WhatsApp AnyTime Best Su...
Hi Fi * Surat ℂall Girls Surat Dumas Road 8527049040 WhatsApp AnyTime Best Su...
 
Benefits of Chanting Hanuman Chalisa .pdf
Benefits of Chanting Hanuman Chalisa .pdfBenefits of Chanting Hanuman Chalisa .pdf
Benefits of Chanting Hanuman Chalisa .pdf
 
Mgr university bsc nursing adult health previous question paper with answers
Mgr university  bsc nursing adult health previous question paper with answersMgr university  bsc nursing adult health previous question paper with answers
Mgr university bsc nursing adult health previous question paper with answers
 
Anti viral drug pharmacology classification
Anti viral drug pharmacology classificationAnti viral drug pharmacology classification
Anti viral drug pharmacology classification
 
Failure to thrive in neonates and infants + pediatric case.pptx
Failure to thrive in neonates and infants  + pediatric case.pptxFailure to thrive in neonates and infants  + pediatric case.pptx
Failure to thrive in neonates and infants + pediatric case.pptx
 
JOURNAL CLUB PRESENTATION TEMPLATE DOCUMENT
JOURNAL CLUB PRESENTATION TEMPLATE DOCUMENTJOURNAL CLUB PRESENTATION TEMPLATE DOCUMENT
JOURNAL CLUB PRESENTATION TEMPLATE DOCUMENT
 
CAD CAM DENTURES IN PROSTHODONTICS : Dental advancements
CAD CAM DENTURES IN PROSTHODONTICS : Dental advancementsCAD CAM DENTURES IN PROSTHODONTICS : Dental advancements
CAD CAM DENTURES IN PROSTHODONTICS : Dental advancements
 
Report Back from SGO: What’s the Latest in Ovarian Cancer?
Report Back from SGO: What’s the Latest in Ovarian Cancer?Report Back from SGO: What’s the Latest in Ovarian Cancer?
Report Back from SGO: What’s the Latest in Ovarian Cancer?
 
Tips and tricks to pass the cardiovascular station for PACES exam
Tips and tricks to pass the cardiovascular station for PACES examTips and tricks to pass the cardiovascular station for PACES exam
Tips and tricks to pass the cardiovascular station for PACES exam
 
Gross Anatomy and Histology of Tongue by Dr. Rabia Inam Gandapore.pptx
Gross Anatomy and Histology of Tongue by Dr. Rabia Inam Gandapore.pptxGross Anatomy and Histology of Tongue by Dr. Rabia Inam Gandapore.pptx
Gross Anatomy and Histology of Tongue by Dr. Rabia Inam Gandapore.pptx
 
Overview on the Automatic pill identifier
Overview on the Automatic pill identifierOverview on the Automatic pill identifier
Overview on the Automatic pill identifier
 
Histopathological staining techniques used in liver diseases
Histopathological staining techniques used in liver diseasesHistopathological staining techniques used in liver diseases
Histopathological staining techniques used in liver diseases
 
Negative Pressure Wound Therapy in Diabetic Foot Ulcer.pptx
Negative Pressure Wound Therapy in Diabetic Foot Ulcer.pptxNegative Pressure Wound Therapy in Diabetic Foot Ulcer.pptx
Negative Pressure Wound Therapy in Diabetic Foot Ulcer.pptx
 
Hemodialysis: Chapter 1, Physiological Principles of Hemodialysis - Dr.Gawad
Hemodialysis: Chapter 1, Physiological Principles of Hemodialysis - Dr.GawadHemodialysis: Chapter 1, Physiological Principles of Hemodialysis - Dr.Gawad
Hemodialysis: Chapter 1, Physiological Principles of Hemodialysis - Dr.Gawad
 
Stereochemistry & Asymmetric Synthesis.pptx
Stereochemistry & Asymmetric Synthesis.pptxStereochemistry & Asymmetric Synthesis.pptx
Stereochemistry & Asymmetric Synthesis.pptx
 
duus neurology.pdf anatomy. phisiology///
duus neurology.pdf anatomy. phisiology///duus neurology.pdf anatomy. phisiology///
duus neurology.pdf anatomy. phisiology///
 

AAOS 2022 Pediatric Questions Answers and Discussion.pptx

  • 2. Figures 1 and 2 are the standing radiographs of a healthy and active 13-year-old female who is being evaluated for an asymmetric appearance of her back. She has no back pain or neurological symptoms. Her first menstrual cycle was 3 months ago. Physical examination shows appropriate motor and sensory strength and no neurological deficits. Based on the patient's age, skeletal maturity, and curve magnitude, what is the most appropriate treatment? A. Physical therapy to work on core strengthening and postural exercises B. Customized rigid thoracolumbosacral orthosis (TLSO) C. Posterior spinal fusion surgery D. Growing rod instrumentation
  • 3. Figure 1 is the standing leg length radiograph of a 4-year-old female who is being evaluated for tibial bowing that has worsened over the past year. Physical examination shows significant genu varum, short stature, and internal tibial torsion. What is the best next step in management? A. Observation with repeat radiographs in 6 months B. Blood work and referral to pediatric endocrinology C. Bilateral lateral proximal tibia hemiepiphysiodesis D. Bilateral proximal tibia osteotomy
  • 4. Figure 1 is the anteroposterior radiograph of the pelvis of a 12-year- old male who weighs 98 kg and has had right knee and thigh pain for the past 4 weeks. He has no history of recent trauma and has been able to ambulate independently with a limp. What is the most common physical examination finding consistent with this pathology? A. Positive Galeazzi sign indicating a leg length discrepancy B. Foot progression angle that is rotated internally with gait C. Increased external rotation when flexing the hip to 90° D. Painful crepitus with hip range of motion
  • 5. A 3-week-old infant is being evaluated for the bilateral foot abnormalities shown in Figure 1. A routine course of weekly Ponseti casting is initiated. Following 4 to 6 weeks of serial casting, what is the most common next step in treatment? A. Percutaneous or open Achilles tenotomy B. Application of full- time bilateral foot abduction orthoses C. Open posterior tibial tendon transfer D. Split anterior tibial tendon transfer
  • 6. Figures 1 and 2 are the radiographs of an 11-year-old girl who is having right elbow pain after “trying to beat up a snowman.” She cannot extend her elbow, has point tenderness to palpation over the proximal ulna. Her underlying condition is associated with a mutation in which gene? A. Fibroblast growth factor receptor 3 (FGFR3) B. Diastrophic dysplasia sulfate transporter (DTDST) C. COL1A1, COL1A2 D. COL2A1
  • 7. Figure 1 is a full standing leg length radiograph of a 15-year-old competitive ski racer who has a small imbalance in his pelvis. He does not have pain or discomfort. Medical history includes a left femur fracture at 2 years of age. Physical examination shows a positive Galeazzi sign with the left femur longer than the right. There is a 1.5-cm leg length discrepancy with the left femur being longer than the right femur. What is the most appropriate treatment? A. Immediate distal femoral and proximal tibial epiphysiodesis of the left leg B. Use of an accommodative shoe lift C. Right femur lengthening with external fixation or an internal magnetic rod D. Left femur shortening osteotomy
  • 8. A 14-year-old female cross country athlete has new onset of left ankle pain and swelling. She has no history of recent trauma. The anteroposterior radiograph of the ankle (Figure 1) shows a small lesion in the superior medial dome of the talus, and the coronal ankle MRI scan (Figure 2) shows greater detail of the lesion. What is the best next step in treatment? A. Custom orthotic and activity modification B. Non-weight bearing in a cast or boot followed by physical therapy C. Open curettage and osteochondral graft implantation D. Ankle arthroscopy with either retrograde or antegrade drilling of the lesion
  • 9. An olecranon fracture in a child is most likely the first presenting sign of which of the following diseases? A. Ehlers-Danlos B. Osteogenesis imperfecta C. Multiple epiphyseal dysplasia D. Osteopetrosis
  • 10. An 8-year-old male sustained the fracture shown in the radiograph in Figure 1. What long-term problem is this child most likely to experience? A.Genu varum B. Genu recurvatum C. Procurvatum D.Genu valgum E.
  • 11. A 2-week-old infant is being evaluated for the foot deformity shown in Figures 1 and 2. Which of the following is the most appropriate course of action? A. Perform serial casting B. Obtain a radiograph of the leg and schedule follow-up C. Reassure the parents that there is no need for further follow-up D. Prescribe physical therapy for stretching exercises and taping
  • 12. In addition to shoulder dystocia and mechanically assisted delivery, which of the following is another common risk factor for brachial plexus birth palsy? A.Oligohydramnios B. Multigravida mother C. Single birth cesarean delivery D.Induced labor
  • 13. A 10-year-old male sustained an angulated radial neck fracture. Closed reduction was not successful, and the patient underwent percutaneous reduction using a Kirschner wire (Figure 1). Injury to the nearby nerve during this procedure is most likely to cause the inability to A. flex the thumb interphalangeal (IP) joint and index finger distal interphalangeal (DIP) joint. B. extend the wrist. C. abduct and adduct the fingers. D. extend the thumb IP joint.
  • 14. Figures 1 and 2 are the radiographs of a 2-month-old male infant who is unable to use his arm. The patient is afebrile and does not appear to be ill. Physical examination of the elbow shows edema, and attempts at passive range of motion elicit pain. Results of laboratory studies are within normal limits. What is the best next step in management? A. Obtain an MRI of the arm B. Obtain a skeletal survey and social work consult C. Apply a splint to the elbow and schedule follow-up D. Aspirate the elbow joint and send for gram stain and culture
  • 15. A 16-year-old male is transferred to the emergency department via ambulance on a backboard and in a cervical collar after he is involved in a rollover motor vehicle collision as a restrained passenger. He says he has low back pain. Vital signs are within normal limits. CT scans of the chest and abdomen are shown in Figures 1 through 4. Which of the following commonly-associated injuries is most likely to require urgent intervention in this patient? A. Duodenum B. Spinal cord C. Liver D. Iliac artery
  • 16. Following a collision with another player, a 16-year-old female soccer player has pain in the right knee and is brought to the emergency department. Physical examination shows exquisite tenderness over the anterior aspect of the knee with difficulty standing. The patient can actively extend the knee to -10° of extension. No effusion or ligamentous laxity is noted. Radiographs are shown in Figures 1 through 3. What is the most appropriate treatment? A. Open reduction and internal fixation of the fracture B. Excision of the bone fragment C. Release of the lateral retinaculum D. Immobilization and nonsteroidal anti- inflammatory drugs (NSAIDS)
  • 17. A 13-year-old baseball pitcher who is left-hand dominant comes to the clinic because he has had pain in his left shoulder for the past 3 weeks. His pitch counts have been <85 pitches on his pitching days with 2+ days of rest between outings. Additionally, he has been working with a pitching coach and practicing with his father who says that he has a very good curve ball. Physical examination shows full range of motion with tenderness to palpation over the proximal humerus. Radiographs show a widened proximal humeral physis. Repetition of what phase of the throwing motion is the most common cause of the symptoms in this patient? A. Stride B. Late cocking C. Acceleration D. Follow through
  • 18. Figure 1 is the radiograph of a 14-year-old female who has had forefoot pain for the past 3 months. She is otherwise healthy. What is the most appropriate initial treatment? A. Metatarsal osteotomy B. Excision of the fragment C. Interposition arthroplasty D. Immobilization in a cast or boot
  • 19. Figures 1 and 2 are the radiographs of a 2-year-old female who is brought to the emergency department because she has had pain and swelling in the right knee since she fell 7 days ago. The parents say she had a bruise over her knee since the fall, but this morning, she awoke with increased pain over the lower leg and was limping. Temperature is 39.0°C (102.2°F), pulse rate is 180/min, respirations are 50/min, and blood pressure is 80/50 mmHg. The patient is very somnolent. On physical examination, discomfort is noted with movement of the right leg. Edema and warmth of the leg are noted. Laboratory studies show a white blood cell count of 22,000/mm3 with 59% polymorphonuclear leukocytes and 25% bands; erythrocyte sedimentation rate is 58 mm/hr; serum C-reactive protein level is 5 mg/L; and platelet count is 443,000/mm3. What is the most appropriate next step in treatment? A. Nonsteroidal anti-inflammatory drugs and immobilization in a long leg cast B. Outpatient MRI and analgesia C. Emergent MRI and admission to the ICU D. Outpatient MRI and oral antibiotics
  • 20. A 12-year-old female basketball player has had bilateral knee pain for the past 2 years. She has had no formal treatment. Physical examination shows a Beighton score of 6/9. Internal rotation of the hip is to 90°, and external rotation of the hip is to 15°. No ligamentous laxity is noted in the knee. Thigh-foot angle is to 30° of external rotation. Examination of the gait shows a normal foot progression angle with kissing patella. What is the most appropriate next step in treatment? A. CT of the knee B. Physical therapy and nonsteroidal anti-inflammatory drugs C. Femoral osteotomy D. Femoral and tibial osteotomy
  • 21. Figure 1 is the radiograph of a 4-year-old girl who is being evaluated for genu varum. She has a family history of bowed legs and short stature. She has a mutation in the PHEX gene. Identify the laboratory studies most consistent with this diagnosi A. Decreased phosphorus, increased serum alkaline phosphatase, normal calcium and vitamin D 25-OH B. Decreased phosphorus and calcium, increased serum alkaline phosphatase and increased PTH, decreased 1,25 OH vitamin D C. Increased phosphorus, increased calcium, decreased alkaline phosphatase D. Increased phosphorus, decreased calcium, increased alkaline phosphatase, and increased PTH
  • 22. In the preoperative planning for the femoral lengthening of a patient with the condition shown in Figure 1, evaluating which of the following during the physical examination is critical in anticipating postoperative complications? A. Hip B. Knee C. Ankle D. Gait
  • 23. A 16-year-old elite ballerina has had pain in the left posterior ankle for the past 6 months. She dances 5 to 6 hours a day, 6 days a week. She says she has the most discomfort when she is en pointe. The patient has no history of injury. Nonsteroidal anti- inflammatory drugs and taping have not provided relief. Physical examination shows pain over the posterior medial ankle with some edema and pain with passive range of motion of the great toe. The heel inverts with toe rise. Radiographs show an os trigonum but no other abnormalities. Which of the following is the most likely diagnosis? A. Posterior tibial tendonitis B. Flexor digitorum communis tendonitis C. Plantar fasciitis D. Posterior ankle impingement syndrome
  • 24. A 2-year-old African American male who walked at 11 months of age presents with bowed legs. His mother says that he is very active and has no limitations, but he commonly trips and falls. Body mass index is within normal limits for his age. On physical examination, the thigh-foot angle is to 30° of internal rotation on the left and to 20° degrees on the right. Coverup test shows minimal genu varum on the left but none on the right. Gait evaluation shows a moderate lateral thrust on the left in the stance phase but not on the right. Standing radiographs with the patellae forward show a metaphyseal-diaphyseal angle of 11° without beaking. What part of this patient's examination is most concerning for infantile Blount disease? A. Body mass index is 21kg/m2 B. Thigh-foot angle C. Gait examination findings D. Radiographic findings
  • 25. A 9-month-old infant is being evaluated for nonaccidental trauma in the emergency department. He is sent for a skeletal survey. What fracture pattern has the highest specificity for nonaccidental trauma? A.Distal humerus physeal separation B. Linear skull fracture C. Proximal tibia corner fracture D.Clavicle fracture
  • 26. Figures 1 through 3 are the radiographs of a 12-year-old female who sustained a twisting injury to her left ankle. She has no history of pain prior to the injury, fever, or systemic symptoms. Physical examination shows no skin findings or soft tissue masses. A subsequent MRI scan is shown in Figure 4, and results of a CT-guided biopsy are shown in Figure 5. Based on these findings, what is the most likely diagnosis? A.Ollier disease B. Osteosarcoma C. Multiple hereditary exostosis D.Jaffe-Campanacci syndrome
  • 27. Although many subjective acute somatic concussion symptoms are short-lived, less information is known about the late effects of concussion and repetitive head impact in patients. However, these patients are known to be at increased risk for which of the following? A.Anterior cruciate ligament injury B. Anterior shoulder dislocation C. Migraine headache D.Amnesia
  • 28. A 2-year-old male presents with a 4-day history of fever and leg pain that progressed to an inability to bear weight. Laboratory studies show elevated white blood cell count, serum C-reactive protein level, and erythrocyte sedimentation rate. MRI scans show osteomyelitis of the distal femur with subperiosteal and intraosseous abscesses. The patient undergoes irrigation and debridement including a cortical window to decompress the intraosseous abscess. Cultures grow methicillin-resistant Staphylococcus aureus (MRSA), and intravenous antibiotics are initiated. The patient initially improves, and laboratory studies show decreasing inflammatory markers. He is afebrile but continues to refuse to bear weight. Three days later, evaluation shows fever, increasing serum C-reactive protein level, and increased edema and erythema of the leg. Repeat MRI scans are shown in Figures 1 and 2. The most appropriate next step in management is to A. continue with intravenous antibiotics and observation. B. initiate anticoagulation for deep vein thrombosis (DVT). C. switch to a different antibiotic. D. repeat irrigation and debridement.
  • 29. Figures 1 through 3 are the radiographs of a 7-year-old male who is being evaluated for pain in the right thigh and an episodic limp. He has no history of injury. Body mass index is in the 99th percentile. Physical examination shows a mild limp on the right with fairly symmetric range of motion except for some mildly limited internal rotation. What is the best next step in treatment? A. In situ pinning of both hips B. Percutaneous pinning of the right hip C. Physical therapy and follow-up in 6 weeks D. In situ pinning of the right hip with prophylactic pinning of the left hip
  • 30. A 3-year-old male is brought to the emergency department because he has had a fever and has refused to bear weight during the past 2 days. Current temperature is 39.0°C (102.2°F). Laboratory studies show a white blood cell count of 13,000/mm3 with a left shift, serum C-reactive protein level of 10 mg/L, and erythrocyte sedimentation rate of 50 mm/hr. Results of blood cultures are pending. Plain radiographs show no abnormalities. MRI scanning is unavailable for the next 2 days. Ultrasonography of the hip is shown in Figure 1. What is the most appropriate next step in treatment? A. Initiate a trial of NSAIDs, and discharge to home with strict return precautions B. Obtain a CT scan of the hip and pelvis C. Admit patient, initiate antibiotic therapy, and get the MRI scan in 2 days D. Perform open irrigation and debridement of the hip
  • 31. Figures 1 and 2 are the clinical photograph and preoperative radiograph of an 18-month-old male with syndromic scoliosis, and Figure 3 is the intraoperative radiograph after application of a Mehta cast. What is the primary benefit of treating this patient's scoliosis with a Mehta cast? A. There is no indication for Mehta casting in this diagnosis B. Can delay the need for future spinal surgery C. Curative treatment for the non-idiopathic scoliosis D. Decreases the incidence of postoperative pneumonia
  • 32. Figures 1 and 2 are the standing radiograph and MRI scan of a 6- year-old female with congenital femoral deficiency. On physical examination, the pelvis levels with a 3-cm block under the short leg. Lachman testing of the knee on the short side shows a soft endpoint with a grade 2 laxity. If this patient undergoes lengthening of the short leg, her risk of what unique complication is most likely to increase? A. Failure of distraction B. Nonunion C. Device breakage D. Knee joint subluxation
  • 33. Figures 1 through 4 are the radiographs and MR arthrogram images of a 12-year- old male who is being evaluated for noticeably decreased range of motion of the right elbow. The patient says that he started having right elbow pain almost a year ago after he played in a dodgeball league with lots of side-arm throws. Physical examination of the right elbow shows a significant lack of flexion (about 10°) and lack of extension (approximately 30°) compared with the contralateral side. Range of pronation and supination is full. No tenderness to palpation is noted at the medial or lateral epicondyles, radial head, or olecranon. No varus-valgus instability is noted. What is the most appropriate course of action? A. Refer the patient to his pediatrician for management B. Initiate an aggressive physical therapy and return-to-sport pathway C. Immobilize in a long arm cast to allow healing of the capitellar lesion D. Order laboratory studies to determine if there is an underlying systemic disease
  • 34. What is a strong predictive factor for femoral head osteonecrosis that can be used as a screening tool in patients with sickle cell disease (HbSS)? A.Body mass index B. Fetal hemoglobin C. Glucocorticoid use D.Ratio of hemoglobin to hematocrit
  • 35. Figure 1 is the radiograph of a 14-year-old male who is right-hand dominant and who fell while snowboarding. On physical examination, the skin is intact, and neurovascular examination shows no deficits. What is the most likely outcome of nonoperative management of this injury? A. Fracture union B. Poor functional outcome C. Faster return to activities D. Symptomatic malunion
  • 36. When compared with intravenous (IV) antibiotics via a peripherally inserted central catheter, postdischarge treatment of pediatric acute osteomyelitis with oral antibiotics is associated with a A.lower rate of treatment failure. B. higher rate of treatment failure. C. lower rate of rehospitalization or return visit to the emergency department. D.higher rate of rehospitalization or return visit to the emergency department.
  • 37. A 13-year-old hockey player sustains a concussion during a play. Physical examination on the sideline shows a loss of consciousness for more than 1 minute, bloody otorrhea, and eyelid hematoma. What is the most appropriate next step in management? a) Emergency transportation to a hospital b)Neuropsychologic testing to identify cognitive deficits c) Return to home and follow-up with a neurologist within 3 days d)Return to home for physical rest and decreased cognitive activity in the immediate 24 to 72 hours
  • 38. Figures 1 and 2 are the radiographs of a 13-year-old male with a 6- month history of left hip pain and a limp. Based on these findings, an abnormality in which of the following serum values is most likely? A. Calcium B. N-telopeptide C. Luteinizing hormone D. Leptin
  • 39. Physical examination of a 2-week-old newborn shows dislocation of the hip associated with developmental dysplasia of the hip. Ultrasonography confirms the clinical examination findings. What factor is most likely to contribute to the successful outcome of Pavlik harness treatment for this patient? A.Female gender B. Hip is dislocated at rest but reducible C. Hip is classified as a Graf type IV on ultrasonography D.A right sided dislocation
  • 40. A 14-month-old toddler is diagnosed with early onset scoliosis. Genetic workup is negative for any syndrome, and radiographs do not show any congenital vertebral malformation. What radiographic finding is most predictive of spontaneous resolution of the curve? A.Curve magnitude at presentation of >25 B. Phase 2 rib changes C. Rib vertebral angle difference of <20 D.Presence of double curves
  • 41. Figures 1 and 2 are the radiographs of a 4-year-old male who is being evaluated for mild pain and swelling along the left tibia as well as occasional limping. Histology of a biopsy of the lesion is consistent with curvilinear trabeculae of woven bone with osteoblastic rimming. What is the most appropriate management? A.Observation B. Curettage and bone grafting C. Wide extraperiosteal excision and reconstruction D.Wide extraperiosteal excision and chemotherapy
  • 42. A 17-year-old lineman sustained an injury to his right foot when another player fell onto his heel while his foot was planted on the ground. After the incident, the patient could not put pressure on his right foot. Physical examination shows significant tenderness to palpation of the midfoot. Plantar ecchymosis is noted. Non-weight bearing radiographs show a subtle gap at the base of the first and second metatarsals. MRI scans are most likely to show an injury to the ligament between the base of the A. first and second metatarsals. B. first metatarsal and medial aspect of the middle cuneiform. C. second metatarsal and lateral aspect of the medial cuneiform. D. second metatarsal and medial aspect of the middle cuneiform.
  • 43. A 5-year-old girl with X-linked dominant hypophosphatemic rickets is evaluated in the clinic for lower extremity deformities. Which of the following best describes the pathology of the defective bone mineralization with this disorder? A.Poor intestinal phosphate absorption secondary to a defect of 1 hydroxylation of 25-hydroxyvitamin D B. Increased phosphate excretion through kidneys secondary to increased fibroblast growth factor 23 level C. Increased phosphate excretion through kidneys secondary to proximal tubulopathy D.Decreased levels of phosphate due to dysfunction of alkaline phosphatase enzymes
  • 44. A 12-year-old male is admitted to the hospital for treatment of a stable slipped capital femoral epiphysis on the left. What radiographic finding is most likely to help the physician to determine if the contralateral hip is at increased risk for subsequent slip? A.Modified Oxford score ‚â•20 B. Superior epiphyseal extension ratio >0.71 C. Epiphyseal-diaphyseal angle difference >21¬∞ D.Posterior epiphyseal extension angle >10¬∞ and open triradiate cartilage
  • 45. A 7-year-old male is evaluated in the emergency department because he has an isolated midshaft fracture of the tibia with an intact fibula. It is most appropriate for the clinician to tell the patient's family that this fracture pattern has a high tendency for what specific deformity? A.Shortening B. Varus C. Malrotation D.Recurvatum
  • 46. Figures 1 and 2 are the radiographs of a premenarchal 12-year-old female with adolescent idiopathic scoliosis. The Cobb angle of the main thoracic curve measures 37°. A rigid thoracolumbosacral orthosis is prescribed for treatment. What is the minimum number of hours that this patient should wear the brace each day to prevent surgery due to curve progression? A. 18 B. 12 C. 10 D. 6
  • 47. Figures 1 through 3 are the radiographs and MRI scan of the left tibia of a 15- year-old male who comes to the emergency department because he has pain and the inability to bear weight on the left side. His temperature is elevated. Laboratory studies show a serum C-reactive protein level of 390 mg/L and an erythrocyte sedimentation rate of 100 mm/hr. Blood cultures confirm a diagnosis of methicillin-resistant Staphylococcus aureus (MRSA), and the patient undergoes multiple surgical debridements to clear the infection. What factor is most likely responsible for the bone and soft tissue toxicity seen during infections with this offending organism? A. Alkaline phosphatase B. Matrix metalloproteinases C. Panton-Valentine leukocidin D. Parathyroid hormone-related protein
  • 48. Following a difficult vaginal birth, a 2-week-old male is being evaluated for Erb palsy. The mother is also concerned about swelling in the right leg and left arm that has been present since birth. Figures 1 through 3 are the radiographs of the right leg and left arm. The patient has not had any associated fevers, and there is no suspicion of nonaccidental trauma since discharge from the hospital. Laboratory studies show elevation of erythrocyte sedimentation rate, serum alkaline phosphatase level, and serum C-reactive protein level. Which of the following other locations in the body most frequently displays the same radiographic findings and is pathognomonic for this diagnosis? A. Metatarsals B. Iliac wing C. Vertebra D. Mandible
  • 49. A 9-year-old male presents with a history of fevers and a limp. Current temperature is 38.9°C (102.0°F). Laboratory studies show an erythrocyte sedimentation rate of 100 mm/hr, white blood cell count of 18,200/mm3, and serum C-reactive protein level of 393 mg/L. An MRI scan shows a large subperiosteal abscess around the right tibia and a large effusion in the ankle. The patient immediately undergoes irrigation and debridement of both areas, which is repeated several times over the next few days. Eventually, the patient is discharged to home. Two weeks later, the patient has development of a venous thromboembolism (VTE) that is treated with anticoagulation. Which data point in this patient's workup is the strongest predictor of the development of a VTE? A. White blood B. C-reactive protein C. Erythrocyte sedimentation rate D. Temperature
  • 50. A 7-year-old female is brought to the emergency department because she has a temperature to 38.8°C (101.8°F), has pain in the right knee, and refuses to walk or crawl. Physical examination shows no tenderness with moderate passive range of motion of the knee. Laboratory studies show an erythrocyte sedimentation rate of 120 mm/hr and a serum C-reactive protein level of 250 mg/L. Aspiration of the right knee yields cloudy fluid with a cell count of 60,000/mm3. When comparing the application of the Kocher criteria to a case of presumed septic arthritis of the knee to the application of the criteria to a case of septic arthritis of the hip, the Kocher criteria A. yields a decreased sensitivity. B. are more sensitive when the WBC count is < 8,000/mm3. C. is less sensitive when C-reactive protein values are added. D. is indistinguishable.
  • 51. A 4-year-old girl who has midlumbar myelomeningocele and hydrocephalus is being evaluated for her uneven gait. A hip examination reveals a right hip dislocation. Bilateral hips are pain- free with full range of motion and no fixed deformity. What is the most appropriate course of action? A. Obtain standing AP pelvis to evaluate the hips; plan open reduction of the right hip when the patient starts having hip pain B. Obtain supine AP pelvis to evaluate the hips; plan open reduction of the right hip as soon as possible C. Inform the family that the dislocated hip will not limit the ability to ambulate and will not benefit from reduction D. Inform the family that the patient's ambulatory potential is poor due to the hip dislocation, which cannot be treated successfully
  • 52. What clinical finding is more likely in pediatric patients with methicillin-resistant Staphylococcus aureus (MRSA) osteomyelitis than methicillin-sensitive Staphylococcus aureus (MSSA) osteomyelitis? A.Concurrent septic arthritis B. Deep vein thrombosis C. Multifocal osteomyelitis D.Subsequent endocarditis
  • 53. A 16-year-old football player is being evaluated because he has groin pain with internal rotation and flexion of the hip. An MRI scan of the hip shows an acetabular labral tear, and radiographs show an alpha angle of 75°. When counseling the patient and his family about next steps, what is the most appropriate recommendation? A.Arthroscopic labral repair B. Arthroscopic femoroplasty C. Surgical hip dislocation with labral repair D.Physical therapy and anti-inflammatory medications
  • 54. A 14-year-old female sustains an anterior cruciate ligament (ACL) injury while participating in a soccer game. Menstruation began at 12 years of age. When counseling this patient and her family about treatment options, what option has the highest chance of success with good patient outcomes? A.Extra-articular reconstruction with the iliotibial band (ITB) B. Anatomic ACL reconstruction with allograft tissue C. Nonoperative bracing until 16 years of age and then ACL reconstruction D.Anatomic ACL reconstruction with autograft tissue
  • 55. An 11-month-old female is evaluated in the emergency department because she has a midshaft fracture of the femur. The patient has met all developmental milestones but has not yet ambulated. What other findings on history or physical examination are most likely to suggest nonaccidental trauma as the cause of this patient's fracture? A.Bruises and cutaneous markings B. Female sex C. Parents serving in the military D.Paternal grandmother living in the home
  • 56. Figure 1 is the radiograph of a 12-year-old baseball pitcher who has a 3- month history of vague shoulder pain and no history of injury. He says he has no pain at rest but feels lateral-based pain after 10 to 15 throws. Physical examination shows decreased internal rotation with the arm in abduction compared with the contralateral side and 5/5 strength in the rotator cuff with no pain and no apprehension. Speed test elicits some mild discomfort. What is the most appropriate next step in management? A. MRI arthrography to rule out labral pathology B. Switch positions to catcher for the upcoming season C. Complete rest from throwing for 6 weeks and then a graduated return to throwing D. Immediate physical therapy consult to work on throwing mechanics and rotator cuff strengthening
  • 57. A fully immunized 16-month-old female is being evaluated because she has irritability and pain in the right hip. She refuses to bear weight on the leg. Temperature is 37.9°C (100.3°F). On physical examination, the hip is held in flexion and external rotation, and pain is elicited with rotational movements. Laboratory studies show a white blood cell count of 10,900/mm3, erythrocyte sedimentation rate of 18 mm/hr, and serum C-reactive protein level of 16 mg/L. Ultrasonography shows a small effusion, and analysis of synovial fluid shows white blood cells of 80,000/mm3 with 88% neutrophils. Immediate irrigation and debridement is performed, but cultures obtained previously and intraoperatively do not grow a definitive organism. What is the most likely diagnosis? A. Septic arthritis with methicillin-resistant Staphylococcus aureus B. Transient synovitis C. Septic arthritis from Haemophilus influenzae type b D. Septic arthritis from Kingella kingae
  • 58. A 1-month-old female is being evaluated because she has not been moving her right arm. Physical examination shows asymmetric pupils, and the right pupil is constricted. No movement of the right upper extremity occurs when the neonatal reflexes are stimulated. It is most appropriate to tell the mother that the outcome of this patient's condition is most likely to be poor because of which of the following reasons? A.There is evidence that the sympathetic chain is involved B. There has not yet been a return of biceps function C. There is evidence of postganglionic involvement D.There is evidence of Erb palsy
  • 59. A 16-year-old high school football player sustains a first- time anterior glenohumeral dislocation during the second game of the season. The patient and his family would like to pursue conservative management with return to play during the same season. Imaging studies do not show a glenoid fracture or large bony Bankart lesion. It is most appropriate to tell the patient and his family that A.bracing of the shoulder has demonstrated lower rates of repeat instability events. B. the risk of recurrent instability in this patient is high. C. the patient will likely be able to return to play in 1 to 2 weeks. D.returning to play during the same interscholastic season is not an option.
  • 60. In a 3-year-old patient, the anterior humeral line crosses the anterior third of the capitellar ossific nucleus with what frequency when compared with the middle third? A.Similar B. Decreased C. Increased D.Variable based on hand dominance
  • 61. A 2-week-old male is brought to the office by his mother because he has had fevers and irritability for the past 24 hours. The patient was born at full term via vaginal delivery with no perinatal complications. Physical examination shows that the right hip is held in a flexed position. The patient cries with attempts to extend the hip. Laboratory studies show a white blood cell A. Repeat ultrasonography of the hip in 1 week B. Place the patient in a Pavlik harness C. Administer antibiotic therapy D. Aspirate the right hip
  • 62. count of 12,400/mm3, erythrocyte sedimentation rate of 44 mm/hr, and serum C-reactive protein level of 76 mg/L. Ultrasonography is shown in Figure 1. What is the most appropriate next step? A. Arthroscopic reconstruction of the anterior cruciate ligament (ACL) B. Physical therapy for range of motion and extension stretches C. Arthroscopic evaluation and internal fixation of the disp
  • 63. Figures 1 through 4 are the radiograph and MRI scans of a 12-year- old female soccer player who has had a large effusion and pain in the left knee after she sustained a noncontact injury 2 days ago. Physical examination shows limited painful motion, diffuse tenderness to palpation medially and laterally, and guarding during Lachman testing. What is the most appropriate next step in management? A. Earlier time to union of the fracture B. Delayed time to union of the fracture C. Greater functional improvement at 6 weeks D. Higher malunion rate of the fracture
  • 64. Figures 1 and 2 are the radiographs of a 10-year-old female who tripped while running. A long leg cast is applied, and the patient is encouraged to begin weight bearing as tolerated. What outcome can the family expect to see compared with non-weight bearing in the cast? A. Perform hip capsulotomy and remeasure epiphyseal perfusion B. Observe the patient for appropriate healing and monitor for avascular necrosis (AVN) C. Perform open femoroplasty to prevent secondary impingement D. Remove the screw and reposition the femoral head to improve perfusion
  • 65. Figure 1 is the radiograph of a 10-year-old gymnast who has had left wrist pain for the past 2 months. Physical examination shows no edema, mild tenderness at the distal radial physis, and pain with forced dorsiflexion of the wrist. What is the most appropriate next step in management? A. Casting of the wrist for 6 weeks B. Distal ulnar epiphysiodesis C. Surgical stabilization with smooth pins D. Activity modification and metabolic workup
  • 66. The four anatomic sites with highest risk of concomitant pediatric septic arthritis and osteomyelitis are A.proximal humerus, distal femur, proximal femur, and distal forearm. B. proximal humerus, proximal ulna, proximal femur, and proximal tibia. C. proximal humerus, proximal radius, proximal femur, and distal fibula. D.proximal radius, distal forearm, proximal femur, and distal tibia.
  • 67. Figure 1 is the radiograph of a 12-year-old male who has activity related right knee pain and a painful click with activity. Physical examination shows a small effusion, stable results on Lachman testing, and no patellar apprehension. Pain is elicited with internal tibial rotation moving from flexion to extension. What is the best next step in management? A. MRI of the knee to evaluate lesion stability B. Triple-phase bone scan to evaluate lesion activity C. CT scan to further determine the extent of the lesion D. Immobilization in a long leg cast for 6 weeks then repeat evaluation
  • 68. Figures 1 and 2 are the radiographs of a 10-year-old soccer player who has a 2-day history of limping with the left leg as well as pain in the left ankle after she sustained an inversion injury. Physical examination shows mild edema of the lateral ankle with tenderness to palpation along the distal fibula. No medial tenderness is noted. Results of anterior drawer and talar tilt tests show stability. The best next step in management is A. excision of the symptomatic os subfibulare with the anterior talofibular ligament. B. open reduction and fixation of the epiphyseal avulsion fracture. C. use of a CAM walker boot and weight bearing as tolerated for 3 to 4 weeks. D. MRI to assess ligamentous injury.
  • 69. Figure 1 is the radiograph of a 14-month-old female who is brought to the emergency department because she refuses to bear weight on her leg. What factor is correlated with nonaccidental trauma (NAT) in this injury? A. Patient is ambulatory B. Fracture pattern is comminuted C. Patient age is <2 years D. Health care coverage is by commercial insurance
  • 70. A youth baseball player has recurrence of proximal humeral epiphysiolysis. Physical examination of shoulder motion in this patient is most likely to show A.increased shoulder external rotation. B. decreased shoulder external rotation. C. increased shoulder rotational arc of motion. D.decreased shoulder rotational arc of motion.
  • 71. An 8-year-old male is being treated for acute hematogenous osteomyelitis of the proximal tibia with empiric intravenous antibiotic therapy. After 48 hours, no clinical improvement is noted, and the patient remains febrile. What is the best next step? A.Obtain an MRI of the knee and tibia B. Obtain an ultrasonography of the knee C. Change to a more broad-spe development. Therefore, screening abdominal ultrasonography followed by serial ultrasounds every 3 to 4 months until age 7 to monitor for tumor development is recommended.
  • 72. A 5-year-old female with Beckwith-Wiedemann syndrome is being evaluated for unequal leg lengths. Physical examination and radiographs show a 1-cm leg length discrepancy and increased girth of the left leg. What diagnostic study is the most appropriate next step? A. Abdominal ultrasonography B. MRI of the lower extremities C. Hip ultrasonography D. MRI of the brain
  • 73. In children with Erb palsy, shoulder contracture and joint deformity develop secondary to weak A.external rotators and abductors. B. internal rotators and abductors. C. external rotators and adductors. D.internal rotators and adductors.
  • 74. Figures 1 and 2 are the radiographs of an 11-year-old male who fell off a trampoline and sustained a Salter-Harris type II fracture of the distal femur. Which parameter is an independent risk factor for the need of additional surgeries to treat associated leg length or angular deformities of the knee that may subsequently develop? A.Type of instrumentation used B. Sex of patient C. Age of patient D.Ossification of the tibial tuberosity
  • 75. Figure 1 is the lateral radiograph of the right calcaneus of an 11- year-old male soccer player who has had pain in the heel for the past 6 weeks that has limited his ability to play. He plays on 2 travel teams, and 1 day a week the patient works with a fitness coach who thinks he needs more plyometric training. On physical examination, the skin is intact. No evidence of neurovascular compromise is noted. Pain is elicited when the calcaneus is squeezed. What is the most appropriate next step in treatment? A. CT to rule out a tarsal coalition B. Injection of the calcaneal apophysis with platelet rich plasma C. Rest, ice, use of heel pads, physical therapy, and nonsteroidal anti- inflammatory drugs D. Drilling the calcaneal apophysis at 4 sites with a smooth 2-mm Kirschner wire
  • 76. A 12-year-old female who is 3 months post menarche is being evaluated for spinal curvature. A detailed neurological examination shows no abnormalities. Radiographs of the spine (Figures 1 and 2) show an apex left lumbar curve measuring 36° and an apex right thoracic curve measuring 33°. What is the most appropriate treatment? A. Thoracolumbosacr al orthosis B. Posterior spinal fusion C. Growing rods D. Observation
  • 77. A 6-year-old male sustains a closed midshaft tibia and fibula fracture after falling off of a trampoline. The patient undergoes closed reduction in the emergency department, and post-reduction alignment is acceptable. At follow-up 1 week later, the fracture has maintained reduction with immobilization in a long leg cast. The parents want to know when their son can start walking on the cast. Based on the literature, early weight bearing for pediatric tibia and fibula fractures is most likely to result in A. earlier return to activity. B. higher rates of malunion at final follow-up. C. fracture displacement and higher rates of surgical intervention. D. decreased fracture healing time with no increase in complications.
  • 78. Figure 1 is the radiograph of a 9-year-old male who has persistent pain in the left upper arm with activity. Medical history includes a fracture of the proximal humerus 1 year ago. What is the most likely diagnosis? A. Aneurysmal bone cyst B. Telangiectatic osteosarcoma C. Little league shoulder D. Unicameral bone cyst
  • 79. A 13-year-old competitive female gymnast has had bilateral elbow pain for the past 6 months. She says she feels clicking and popping in the elbow. Radiographs are shown in Figures 1 and 2, and Figure 3 is an MRI scan that shows loose cartilage bodies in the joint. What is the most appropriate treatment? A. Cessation of gymnastics activity B. Elbow arthroscopy with loose body removal and associated microdrilling C. Open reduction and internal fixation of the osteochondral fragment D. Physical therapy to work on elbow and shoulder range of motion and strengthening exercises
  • 80. A 12-year-old male pitcher who is right-hand dominant is being evaluated for pain in the anterior lateral shoulder that is associated with pitching. The pain is severe in the late cocking phase. The patient pitches on 2 separate baseball teams year round and has a private pitching instructor. What finding is most likely on MRI scan of this patient? A. Labral and rotator cuff pathology B. Proximal humeral physeal narrowing C. Metaphyseal edema adjacent to the physis D. Stress fractures in the proximal humeral shaft
  • 81. Figure 1 is the radiograph of a 13-year-old male basketball player who has a 3-month history of throbbing pain over the anterior knee with activity. He plays basketball on his school team and on a competitive club team. He also runs track to improve his vertical jump. Physical examination shows tenderness to palpation at the tibial tubercle. What is the most appropriate treatment? A. Rest, ice, nonsteroidal anti- inflammatory drugs, and stretching B. Surgical excision of the fragmented apophysis ossicles C. Injection of platelet rich plasma D. MRI of the knee to evaluate for interarticular pathology
  • 82. An 8-year-old male is admitted to the hospital for treatment of distal femur osteomyelitis with a subperiosteal abscess. Culture of the abscess is positive for methicillin-resistant Staphylococcus aureus. What role does coagulase play in this patient's musculoskeletal infection? A. Adherence of Staphylococcus aureus to fibrin protecting it from phagocytosis B. Activation of prothrombin to thrombin leading to clot and abscess formation C. Lysis of neutrophils due to pore formation at the cell surface D. Activation of platelets and bacterial adhesion to blood vessel walls
  • 83. An otherwise healthy 2-year-old female is brought to the emergency department because she has refused to bear weight on the right leg for the past 2 days. The patient has no history of trauma, and she attends day care. Temperature is 38.0°C (100.4°F). On physical examination, pain is elicited on movement of the right hip. On laboratory studies, serum C-reactive protein level is 16 mg/L, and white blood cell count and erythrocyte sedimentation rate are within normal limits. An MRI scan of the right hip shows proximal femoral osteomyelitis with surrounding joint effusion. The patient undergoes irrigation and debridement of the right hip and proximal femur. In addition to aerobic bacterial cultures, what other test is most appropriate to identify the most likely causative bacteria? A. Fungal culture B. Acid-fast bacteria (AFB) C. Anaerobic bacterial culture D. Polymerase chain reaction (PCR)
  • 84. A 13-year-old male with adolescent Blount disease is undergoing hemiepiphysiodesis with a tension band to correct the mechanical axis. Body mass index is 62 kg/m2. The medial proximal tibial angle for the affected limb is 70°. Which implant is most likely to provide the lowest risk of failure? A. 1 epiphyseal and 1 metaphyseal cannulated titanium screw B. 2 epiphyseal and 2 metaphyseal solid titanium screws C. 1 epiphyseal and 1 metaphyseal cannulated stainless steel screw D. 2 epiphyseal and 2 metaphyseal solid stainless steel screws
  • 85. A 14-year-old female who is skeletally mature is brought to the emergency department after she sustained a high-energy open tibia fracture in a snowmobile accident. Tertiary survey is negative for any additional trauma. Intraoperative findings show compartments that are soft and swollen but compressible. Distal vascularity is intact, and wound edges cannot be reapproximated. The clinical photograph in Figure 1 shows the appearance of a 17-cm open wound following emergent excisional irrigation and debridement and initiation of intravenous antibiotics. In addition to plastic surgical flap coverage in 24 to 48 hours, what is the most appropriate immediate surgical treatment? A. Loose wound reapproximation, sterile dressing application, use of a long leg posterior splint, plan to repeat irrigation and debridement, and intramedullary nailing B. Application of a ring external fixator, vacuum-assisted dressing application, and plan to repeat irrigation and debridement C. Immediate intramedullary nailing, vacuum-assisted dressing application, and plan to repeat irrigation and debridement D. Immediate submuscular plate osteosynthesis, vacuum- assisted dressing application, and plan to repeat irrigation and debridement
  • 86. Figures 1 and 2 are the radiographs of a 6-year-old male who is brought to the emergency department after he sustained an isolated femoral shaft fracture after being struck by a car while riding a scooter. What is the most appropriate treatment? A. Submuscular plating B. Immediate hip spica casting C. Flexible intramedullary nailing D. Locked intramedullary nailing with trochanteric entry
  • 87. Figures 1 and 2 are the sagittal CT scan and sagittal STIR sequence MRI scan of a 15-year-old male who is brought to the emergency department after he sustained an injury to his spine in a high-speed motor vehicle collision. He was a restrained rear-seat passenger and was wearing a lap belt. Physical examination shows hypovolemic shock, a distended abdomen, ecchymosis across the anterior abdomen, and bilateral weakness in quadriceps and ankle dorsiflexion. What is the most appropriate surgical treatment? A. Immediate open reduction, decompression, and fusion of the compression fracture of the lumbar spine B. General surgical exploratory laparotomy followed by staged treatment of the compression fracture of the lumbar spine with an extension thoracolumbosacral orthosis C. General surgical exploratory laparotomy followed by staged instrumented reduction and fusion of the Chance fracture of the lumbar spine D. Immediate instrumented reduction and fusion of the Chance fracture of the lumbar spine followed by general surgical exploratory laparotomy
  • 88. Figure 1 is the radiograph of a 6-year-old male with Down syndrome whose parent says that they can hear a popping in his right hip. Physical examination shows a Trendelenburg gait and a leg length discrepancy. Barlow test is positive in the right hip. What is the most appropriate treatment? A. Adductor longus tenotomy, arthrography, closed reduction, and application of a hip spica cast B. Open reduction and capsulorrhaphy, acetabular osteotomy, and proximal femoral varus osteotomy C. Proximal femoral varus osteotomy only D. Closed reduction and spica casting
  • 89. A 5-year-old patient presents to the emergency department with a right knee effusion, antalgic gait, and limited range of motion of the knee. The patient is afebrile. Physical examination shows moderate limitation of passive range of motion of the knee from 20 to 90° of flexion, and pain is noted at the end points. Laboratory studies show a white blood cell count (WBC) of 13,500/mm3 and erythrocyte sedimentation rate (ESR) of 42 mm/hr. What is the most appropriate next step in treatment? A. Knee aspiration and synovial fluid analysis B. Urgent arthroscopic surgical lavage of the knee C. Admission to the hospital for observation with initiation of intravenous antibiotics D. Discharge to home with oral nonsteroidal anti-inflammatory drugs
  • 90. Figures 1 and 2 are the coronal and sagittal STIR MRI scans of a 7-year-old female who has had worsening pain in the left ankle over the past week and new onset of inability to bear weight on the leg upon awaking this morning. Temperature is 39.0°C (102.2°F). Laboratory studies show a white blood cell count of 15,500/mm3, serum C-reactive protein level of 16.7 mg/L, and erythrocyte sedimentation rate of 50 mm/hr. What is the best next step? A. Arthroscopic lavage of the left ankle B. Surgical irrigation and debridement of the left distal tibia C. CT scan of the chest and an oncology consult D. Empiric intravenous antibiotics and E. serial inflammatory markers to assess response
  • 91. Figure 1 is the radiograph of a 13-year-old basketball player who twisted her ankle during a layup. She says she felt a pop and limped off the court. On physical examination, the skin is intact. Neurovascular examination shows no abnormalities. This patient's injury is most likely caused by avulsion of what ligament? A. Anterior-inferior tibiofibular B. Calcaneofibular C. Anterior talofibular D. Plantar facial ligament
  • 92. A Tillaux fracture occurs when the anterior-inferior tibiofibular ligament avulses an epiphyseal fragment off the anterolateral tibia, typically from an external rotation mechanism. A Salter-Harris type III fracture occurs. This is a common adolescent sports injury. Ankle physeal closure occurs between 12 to 15 years of age. The distal tibia physis closes from medial to lateral, leaving the lateral physis vulnerable to this transitional avulsion fracture. The radiograph shows the typical appearance of this fracture. The anterior talofibular ligament, posterior-inferior tibiofibular ligament, and calcaneofibular ligament are not involved. A. Glascow coma scale. B. Sport Concussion Assessment Tool 5 (SCAT5). C. EyeBoxCNS test. D. Immediate Post-Concussion Assessment and Cognitive Testing (ImPACT) battery.
  • 93. A 12-year-old female who is in eighth grade has had progressive bowing of the left leg over the past 2 years. She has no history of trauma to the leg, and she does not have pain. Weight is 249 lb. On physical examination, the right leg is straight. Radiographs of the left knee are shown in Figures 1 and 2. Adolescents with this disorder also most commonly have what medical condition? A. Vitamin D deficiency B. Amenorrhea C. Hypertension
  • 94. Figures 1 through 4 are the radiographs and MRI scans of a 16-year-old male lacrosse player who has had pain and swelling of the left knee for the past 3 weeks. He has not had fever, weight loss, or change in appetite, and he has no history of the knee giving way. He is afebrile. Laboratory studies show a white blood cell count of 9000/mm3, erythrocyte sedimentation rate of 32 mm/hr, and serum C-reactive protein level of 0.8 mg/L. What is the most likely diagnosis? A. Anterior cruciate ligament (ACL) tear B. Osteosarcoma C. Ewing sarcoma D. Chondroblastoma
  • 95. Figures 1 and 2 are the radiographs of a 15-year-old male who comes to the emergency department because he has had pain and swelling of the right knee since he landed a jump shot during a basketball game. It is most appropriate to also assess this patient for what associated injury? A. Posterior cruciate ligament tear B. Compartment syndrome C. Common peroneal injury D. Fibular fracture
  • 96. An 18-month-old male is being evaluated for a right flatfoot deformity. Physical examination shows a rigid deformity of the foot. Radiographs of the foot are shown in Figures 1 and 2. What is the most appropriate treatment? A. Observation B. Bracing with a supramalleolar orthosis C. Serial casting followed by surgical correction D. Medial column shortening with lateral column lengthening
  • 97. A 16-year-old-high school wrestler is referred by his primary care physician for low back pain that has been present for the past 3 months. Physical examination shows pain with extension of the lumbar spine. The patient cannot touch the toes when asked to bend forward. Neurologic examination shows no focal deficits, and result of straight leg raise test is negative. What is the most appropriate initial imaging study of the lumbar spine? A.Anteroposterior and lateral radiographs B. Anteroposterior, lateral, and oblique radiographs C. CT scan D.SPECT scan
  • 98. Of the following genes that are necessary for normal hindfoot development, which has been implicated as a cause of clubfoot? A. PITX1-TBX4 B. HOX C. FLNBM D. TGFB1
  • 99. A very large 1-week-old female infant presents to the orthopaedic clinic after undergoing a difficult delivery. The parents are concerned that the patient is not moving her right upper extremity as much as the left. On physical examination, the right upper extremity is resting in adduction and internal rotation with the wrist in flexion. Full passive range of motion is noted at the shoulder, elbow, wrist, and digits. The most appropriate treatment is A. immediate brachial plexus nerve transfers. B. a daily regimen of passive range of motion exercises and stretching. C. planned brachial plexus nerve transfers when the patient is 6 to 12 months old. D. planned humeral derotational osteotomy when the patient is 18 months old.
  • 100. Figures 1 through 3 are the clinical photographs of a 4-year-old female who has an abnormal gait. She underwent serial casting as an infant and used boots-and-bar orthotics for about 1 year afterward. The recurrent deformity is passively correctable. What is the most appropriate next step in treatment? A. Use of a boots-and-bar orthosis and refer to physical therapy for gait retraining B. Perform circumferential posteromedial release followed by use of a boots-and-bar orthosis C. Repeat serial casting followed by tibialis anterior tendon transfer D. Repeat serial casting and refer to physical therapy for gait retraining
  • 101. Figure 1 is the radiograph of a 15-year-old male who presents to the orthopaedic office because he has a several-month history of low back pain. He is active in football, wrestling, and weight lifting, and he has no history of trauma or injury. He says the pain is aggravated by prolonged sitting and is relieved by movement and taking nonsteroidal anti- inflammatory drugs. Physical examination shows limited forward flexion. The patient is able to heel-to-toe walk without difficulty. Straight leg raise test does not elicit significant pain. What is the most likely diagnosis? A. Disk herniation B. Spinal cord tumor C. Scheuermann disease D. Thoracolumbar fracture